Series 7 Quiz Questions

Réussis tes devoirs et examens dès maintenant avec Quizwiz!

Prime Brokerage Account

a customer (an institution) selects one member to provide custody and financing of securities and executes trades with other firms known as executing brokers.

. If a member wishes to appeal an adverse decision in a Code of Procedure (COP) hearing, the member must appeal to the National Adjudicatory Council (NAC) within how many days of the decision date? A. 25 B. 30 C. 40 D. 45

A

A corporation has an outstanding issue of 8% convertible debentures with a conversion price of $25. The bond indenture contains an antidilutive clause guaranteeing the debt holders the right to maintain proportionate equity conversion in the corporation. If the company pays a 10% stock dividend to its common shareholders, how will that affect the debenture holders? A) The bonds will now be convertible at approximately 22.73. B) The interest rate on the debentures will increase to 8.8%. C) They will receive four shares of the common stock. D) Each debenture holder will receive a check for $100.

A

A customer places an order to buy 300 DWQ at 140 stop, but not over 140.25. This is A) a buy stop limit order. B) a buy stop order. C) a buy limit order. D) a market not-held order.

A

A new bond issue will include warrants to A) increase the attractiveness of the issue to the public. B) increase the price of the issue to the public. C) increase the spread to the underwriter. D) increase the capital raised by the issuer through the bond offering.

A

A portfolio that invests in blue-chip stocks and growth stocks can best be described as A) a growth and income portfolio. B) a balanced portfolio. C) an aggressive portfolio. D) a high-yield portfolio.

A

A prospectus for a variable annuity contract must provide full and fair disclosure. is required by the Securities Act of 1933. must be filed with FINRA. must precede every sales presentation. A) I and II B) III and IV C) II and IV D) I and III

A

A customer sells 1 ABC Corporation put for 2 on February 22, 2019, with a strike price of 50 and an expiration date of March 16, 2019. On March 15, 2019, ABC is put to the customer. Which of the following statements about this transaction is correct? A) He has an acquisition cost of $4,800 and a date of acquisition of March 15, 2019. B) He has an acquisition cost of $5,000 and a date of acquisition of March 16, 2019. C) He has a $200 short-term gain on the sale of his put. His cost of acquisition is $5,000, and the date of acquisition is February 22, 2019. D) He has an acquisition cost of $4,800 and a date of acquisition of February 22, 2019.

A- $5000 - $200

Hedge Fund Characteristics

- have high minimum initial investment - only available to accredited investors - lack liquidity: several restrictions when trying to redeem

A variable annuity's separate account is: used for the investment of funds paid by contract holders. used to escrow late or otherwise delinquent premium payments. required to be located off of the company's premises. regulated under both securities and insurance laws. A) II and IV B) II and III C) I and IV D) I and III

A

A variable-rate municipal bond investment's main advantage is that A) its price should remain relatively stable. B) its interest is exempt from all taxes. C) it is noncallable. D) it is likely to increase in value.

A

All of the following are true of stockholders' equity except A) that it is carried as an asset on the balance sheet. B) that it is reflected in the book value of the stock. C) that it consists of stock issued, capital surplus, and retained earnings. D) that it is also called net worth.

A

All of the following terms are associated with the holder of an option except A) seller. B) owner. C) long. D) buyer.

A

An employee not covered under his company's pension plan has been contributing to a traditional IRA for five years. If he leaves his current job, starts a new job, and is covered under the new corporation's pension plan, which of the following statements is true? A) Contributions to his traditional IRA may continue. B) Contributions to his IRA must stop; the money in the account will be frozen, but interest and dividends can accrue tax free until he retires. C) The money in his IRA must be combined with any money he will receive from the pension plan. D) His traditional IRA must be closed.

A

Arbitration and mediation are two services provided by FINRA to settle disputes between members. Regarding these services, which of the following statements are not true? Mediation is mandatory; arbitration is not. Arbitration always results in a binding decision; mediation may not. If arbitration is unsuccessful, the dispute moves on to mediation. A mediator in a dispute may not serve as an arbitrator in the same dispute. A) I and III B) I and IV C) II and III D) II and IV

A

As a new registered representative, there is much industry jargon to learn. If you overheard your manager discussing a stock power, it would be in reference to A) good delivery of a stock certificate. B) the firm's technical analyst reporting a breakout on a stock. C) authorization to sell stock in a discretionary account. D) a power of attorney granted from a client to a third party.

A

Before effecting an initial penny stock transaction for a new customer, the registered representative must confirm whether the person is an established customer. obtain a signed risk disclosure document from the customer. obtain a signed suitability statement from the customer. determine suitability based on financial condition, investment experience, and investment objectives. A) II, III and IV B) I, II, III, and IV C) I and IV D) I and II

A

Bondholders may not take action against the corporation if it fails to make interest payments for A) income bonds. B) convertible bonds. C) debentures. D) subordinated debentures.

A

Customer account information must be updated at least every A) 36 months. B) 12 months. C) 48 months. D) 24 months.

A

Danielle is the CFO of the XYZ Manufacturing Company. XYZ's shares are listed on the NYSE. She purchased shares of XYZ common stock through her registered representative of a FINRA member firm 165 days ago and, wishing to add on to her home, sells the stock and realizes a $50,000 profit. Under SEC rules, how will this transaction be treated? A) This is considered a short-swing profit and it must inure to the benefit of the issuer. B) Because the shares were purchased in the open market, she gets to keep the profit. C) This is a violation of the rules on insider trading and could result in fines and/or imprisonment. D) The sale will be reversed and she will have her initial investment returned to her.

A

Each of the following choices are potential sources of funds associated with the backing of a revenue bond issue except A) fines. B) ticket revenues. C) concessions. D) lease payments.

A

FINRA Rule 2232 deals with customer confirmations. Among the requirements in that rule are all of the following except A) a written confirmation of a transaction for a customer must be given or sent to such customer within one business day of the transaction. B) if the broker-dealer is acting as principal, stating whether it is a market maker in the security. C) the confirmation must conform to the requirements of SEC Rule 10b-10. D) a written confirmation of a transaction for a customer must be given or sent to such customer at or before completion of the transaction.

A

Five years ago, a corporation issued a portion of its authorized shares. Those shares currently trade on the New York Stock Exchange. In an effort to reduce the number of shares outstanding, the issuer purchases 30 million shares from existing shareholders. The shares purchased by the issuer in the secondary market are now known as A) treasury stock. B) unissued stock. C) authorized stock. D) issued stock.

A

If a customer is concerned about interest rate risk, which of the following securities is least appropriate? A) 25-year municipal bonds B) 10-year corporate bonds C) Treasury bills D) 5-year corporate bonds

A

If the State of Texas has solicited bids for a proposed municipal bond offering, the underwriters for that offering would be the syndicate that would A) sell the issue at the lowest net interest cost to the State of Texas. B) generate the most proceeds for the State of Texas. C) sell the issue at the lowest price. D) sell the issue at the highest price.

A

In an effort to raise additional capital, which type of registered investment company may issue debt securities? A) A closed-end investment company B) A unit investment trust C) An open-end investment company D) A face amount certificate company

A

In the partnership agreement of a limited partnership, all of the following would be disclosed except A) the procedures for the annual election of general partners. B) how the general partners will be compensated. C) what matters the limited partners can vote on under the democracy provisions. D) how the operating profits will be distributed.

A

On Monday, June 1, an investor pays 92 to purchase a 5% J&J municipal bond maturing on July 1, 2030. Purchasers of bonds pay accrued interest to the seller, in addition to the market price of the bond. How many days of accrued interest will this seller receive? A) 152 B) 153 C) 154 D) 151

A

SEC regulations for securities issued by investment companies prohibit which of the following? Closed-end funds from issuing preferred stock Open-end funds from issuing preferred stock Closed-end funds from issuing bonds Open-end funds from issuing bonds A) II and IV B) I and IV C) II and III D) I and III

A

The amount paid into a defined contribution plan is set by A) the trust agreement. B) the employee's age. C) the ERISA-defined contribution requirements. D) the employer's profits.

A

The performance of the XYZ Growth Fund has been in the top 1% of all funds in its category for the past 1-, 5-, and 10-year periods. Which of the following would be the biggest risk factor to an investor investing in this fund? A) The manager's tenure is six months B) Lack of diversification in the portfolio C) Past performance is no assurance of future results D) A dividend yield of less than 2%

A

There are many different types of asset-backed securities, but the common theme uniting all of them is A) they are supported by a contractual obligation to pay. B) they are a form of equity financing. C) they are usually backed by a single source of payment. D) they tend to be exchange traded.

A

When a broker-dealer makes a market, it is acting as A) a principal. B) a broker. C) an underwriter. D) an agent.

A

Which of the following are a direct obligation of the U.S. government? A) Ginnie Maes B) Government bond mutual funds C) Treasury receipts D) Fannie Maes

A

Which of the following is true regarding an institutional communication? A) An individual with $50 million or more in total assets is considered an institution. B) No more than five retail customers may receive an institutional communication. C) It must be filed with FINRA. D) It must be preapproved by a principal.

A

Which of the following statements about a Coverdell Education Savings Account (ESA) is not true? A) Contributions are tax deductible, subject to a modified adjusted gross income phase out. B) The funds grow income tax deferred and, if used for elementary, secondary, or college educational expenses, the earnings are tax free. C) Contributions of $2,000 per child per year are allowed. D) Contributions can be made to this type of plan and a Section 529 plan in the same year for the same beneficiary.

A

Which of the following statements regarding oil and gas limited partnerships are true? Developmental programs are more risky than exploratory programs. Exploratory programs are more risky than developmental programs. Successful developmental programs provide higher returns than exploratory programs. Successful exploratory programs provide higher returns than developmental programs. A) II and IV B) II and III C) I and III D) I and IV

A

Which of the following types of municipal bonds is subject to statutory debt limits? A) General obligation (GO) bonds B) Industrial development revenue (IDR) bonds C) Special tax bonds D) Hospital bonds

A

Which of the following would not be found in the underwriting of a new corporate bond issue? A) A legal opinion B) A market out clause relieving the underwriter of his responsibility C) A stabilization clause D) The holding of a due diligence meeting

A

n example of overlapping debt would be a school district and A) county general debt. B) a water pollution control facility. C) corporate debt of the county's largest employer. D) a local utility power plant.q

A

Progressive taxes would include personal income tax. gift taxes. estate taxes. excise taxes. A) I, II, and III B) I and II C) II, III, and IV D) I and III

A Progressive taxes are those where the rate of taxation increases as the dollars being taxed increase. Personal income tax, while not as progressive as it was before the 1986 reform, is still considered a progressive tax because the highest tax rate is levied against the highest earnings. Gift taxes and estate taxes are highly progressive, but excise taxes, such as fuel tax and transportation tax, are a fixed rate, and therefore, would not be considered progressive.

If an investor buys a LEAPS contract on issuance and allows it to expire unexercised, what is the investor's tax consequence at expiration? A) Long-term capital loss B) Short-term capital gain C) Long-term capital gain D) Short-term capital loss

A A LEAPS contract has an expiration of more than one year. Upon expiration, the buyer incurs a long-term capital loss equal to the amount of the premium paid.

An investor purchases a newly issued convertible bond at par. The bond is convertible at $40. Three years later, the underlying common stock is trading at $50 per share. If the investor sells the bond at a $50 premium over the parity price, there is A) a long-term capital gain of $300. B) a long-term capital gain of $200. C) a long-term capital gain of $10 per share. D) a long-term capital gain of $1,050.

A A bond convertible at $40 per share has a share conversion rate of 25 shares ($1,000 ÷ $40). The second step is to compute the parity price. That is, what are those 25 shares worth? Multiply 25 shares times $50 per share and that equals $1,250. When the bondholder sells the bonds at parity plus a $50 premium, $1,300 is received. The $300 profit over the $1,000 initial cost is a long-term capital gain.

An investor purchases a bond on its initial public offering. Even though the bond has a maturity value of $1,000 in 10 years, the offering price is only $600. If this investor holds the bond until it matures, A) there is no reported capital gain. B) $400 is reported as ordinary income. C) there is a $400 long-term capital gain. D) there is a $360 long-term capital gain and $40 in ordinary income.

A A bond issued at a significant discount from its maturity value is known as an original issue discount bond (OID). In the case of a corporate bond, the computation is more complex than can be tested, but there are two things you need to know: A portion of the discount is taxed as ordinary income each year until maturity, even though it is not actually received. This is called phantom income. Each year's taxable amount is reported on Form 1099-OID. Because a portion of the discount has been taxed each year, at maturity all $400 of the discount has been accreted. This results in no reported capital gain

Which of the following is not a type of corporate debt instrument? A) Revenue bond B) Mortgage bond C) Income bond D) Subordinated debenture

A A revenue bond is a type of municipal bond

If a customer wishes to buy 1 XYZ option and sell another XYZ option, but he is not willing to spend more than $300, which of the following orders should be entered? A) A spread order B) A straddle order C) Two stop orders D) Two limit orders

A A spread involves the simultaneous purchase and sale of different option contracts of the same type. A spread incurs a gain or loss depending on what happens to the difference in the premiums between the two contracts. Because this investor wants to limit his risk to $300, he would buy the spread at a net debit of $300 or less. (This is one order, not two.)

An employer-sponsored retirement plan that pays a specific benefit to participants at their normal retirement age is A) a defined benefit plan. B) a supplemental employee retirement plan. C) a defined contribution plan. D) a Section 401(k) plan.

A A traditional defined benefit plan promises to pay a specific benefit to a participant at his normal retirement age, as specified by the plan document.

Your customer, age 60, is retired and living at home with a fully paid-off mortgage. Her portfolio contains growth stocks and high-quality bonds, and she is a long-time investor and comfortable with moderate risk. Her objective is a moderate level of current income to supplement her corporate pension plan distributions and the earnings from her traditional IRA. How are the distributions taxed from her IRA? A) All taxable distributions from a traditional IRA are taxed as ordinary income. B) If the IRA has been owned for more than one year, the distributions are long-term capital gains. C) If a security is sold for more than it was purchased for, the distribution of the profit is taxed as a capital gain. D) The distribution is taxed as either ordinary income or capital gains, depending on the source of the distribution.

A All taxable distributions from a retirement account, including IRAs, are taxed as ordinary income, not capital gains.

A characteristic of hedge funds that would not be found in a mutual fund is A) a lock-up period. B) professional management. C) a diversified portfolio. D) the ability to be purchased on margin.

A Although hedge funds can use margin in portfolio transactions, they, like mutual funds, cannot be purchased on margin.

An employee not covered under his company's pension plan has been contributing to a traditional IRA for five years. If he leaves his current job, starts a new job, and is covered under the new corporation's pension plan, which of the following statements is true? A) Contributions to his traditional IRA may continue. B) His traditional IRA must be closed. C) The money in his IRA must be combined with any money he will receive from the pension plan. D) Contributions to his IRA must stop; the money in the account will be frozen, but interest and dividends can accrue tax free until he retires.

A An employee covered under a qualified retirement plan may continue to own and contribute to an IRA. The contributions to a traditional IRA may not be fully tax deductible, depending on the amount of compensation earned, but the employee benefits from the tax deferral of IRA earnings.

On April 15, 2016, your client purchased a variable life insurance policy with a death benefit of $450,000. The November 2019 statement showed a cash value of $28,000. If the client wanted to borrow as much as possible, the insurance company would have to allow a loan of at least A) $21,000. B) $28,000. C) $14,000. D) $25,200.

A At the three-year mark, that minimum becomes 75% of the computed cash value. Seventy-five percent of cash value of $28,000 is $21,000.

An investor purchased a municipal bond at par to yield 5.5% to maturity. If, two years later, she sold the bond at a price equivalent to a 5% yield to maturity, the investor incurred A) a capital gain. B) no taxable result at this time. C) a capital loss. D) taxable interest income.

A Because the investor sold the bond at a price that will yield less than the yield when she purchased the bond, the bond must have been sold for more than the investor paid for it. Therefore, the investor profited by that difference. Remember, the higher the price, the lower the yield.

The RJN Corporation has issued warrants where each warrant offers the holder the right to purchase one share of RJN's common stock for $20 per share. The warrants are exercisable anytime within the next five years. Chelsea purchases 80 warrants for $2 each. If three years after the purchase, the market price of RJN common stock has risen to $25 per share and Chelsea sells the warrants for their intrinsic value, she has realized A) a short-term capital gain of $240. B) a long-term capital gain of $240. C) a short-term capital gain of $400. D) a long-term capital gain of $400.

A Being able to purchase stock at $20 per share when the market price is $25 per share means the warrant is intrinsically worth $5. With 80 warrants, the sale proceeds are $400 (80 times $5 each). Don't forget that Chelsea paid $2 for each warrant, a cost of $160 (80 times $2). That makes her net gain $240 ($400 proceeds minus $160 cost). The gain is long-term because she held the warrants longer than 12 months.

If XYZ common stock has a $4 dividend, a yield of 4.2%, a price-to-earnings (P/E) ratio of 12, and it is trading at $96, its approximate earnings per share (EPS) is A) $8.00. B) $4.00. C) $48.00. D) $50.40.

A Dividing the stock's price by the P/E will give the EPS ($96 / 12 = $8).

Asset allocation refers to the spreading of portfolio funds among different asset classes with different risk and return characteristics. When allocating among asset classes, you would not include A) ETFs. B) bonds. C) stock. D) cash and cash equivalents.

A ETFs are a way of investing in equity (stock) or debt (bonds) securities and are not a separate asset class.

A customer of a member firm has just invested $100,000 into an equipment leasing DPP. Under FINRA rules, the maximum compensation allowable to the firm is A) $10,000. B) $5,000. C) $2,000. D) $15,000.

A FINRA Rule 2310 limits compensation on the sale of a DPP to 10% of the offering price

An investor holds 3,000 shares of a stock with a current market value of $12 per share. After a 1:6 stock split, the investor's position will be A) 500 shares with a market value of $72 per share. B) 15,000 shares with a market value of $12 per share. C) 15,000 shares with a market value of $2.40 per share. D) 500 shares with a market value of $2.40 per share.

A For each share owned, the investor will now have 1/6 of a share. That turns 3,000 shares into 500. At the same time, the market price per share will increase approximately by a factor of six. The key to any stock split question is that the total value of the account remains the same. Presplit, it was 3,000 × $12 = $36,000 and postsplit it is 500 × $72 = $36,000.

Which of the following usually does not pay interest semiannually? A) Government National Mortgage Association (GNMA) B) Treasury bonds C) Public utility bonds D) Treasury notes

A GNMA pass-through certificates pay principal and interest monthly. The others usually pay interest semiannually.

All of the following must register as an investment company under the Investment Company Act of 1940 except A) an initial public offering for common shares of Amalgamated Investments, a holding company. B) a new stock fund created by GHI Mutual Fund Distributors. C) certificates issued by a face amount certificate company. D) an initial public offering for shares of a closed-end management company.

A Holding companies are not included in the definition of investment company under federal law. Amalgamated Investments would register with the SEC, just as any other offering of common stock.

Your client's position is long 100 MNO purchased at 90. Which of the following strategies will limit the customer's loss to $700? A) Long one MNO 90 call at 4, long one MNO 90 put at 3 B) Sell a MNO 90 call at 7 C) Short one MNO 90 call at 4, short one MNO 90 put at 3 D) Buy a MNO 90 call at 7

A If the MNO stock drops to $0, the customer loses $9,000 on the long stock position but retains the right to sell the stock to someone at $9,000, to prevent loss beyond the premium of $300. The call would expire out of the money, for a total loss of $700.

Greater Growth Capital (GGC), a FINRA member firm, has just been acquired by Better Retirement Outcomes (BRO), a much larger FINRA member. If GGC would like to effect a bulk transfer of its customer accounts using a negative consent procedure, FINRA rules A) prohibit GGC from charging a fee to any existing GGC customers who decide to transfer their accounts to a different firm. B) require GGC to obtain affirmative written consent before transferring a customer's account to BRO. C) prohibit GGC from charging a fee to any existing GGC customers who elect to transfer their accounts to BRO, but permit a nominal charge if the customer wishes to transfer to another member firm. D) require that GGC send a notice to each affected customer at least 60 calendar days before it effects the bulk transfer.

A In the case of a bulk transfer, such as when a member firm is acquired by another member, some customers may wish to opt out of the transfer. In those cases, it is prohibited for them to be charged any fees for transferring their accounts to another member firm. This is true only if the customer opts out during the allowable time period, which is 30 calendar days, not 60. The FINRA rule permits negative consent as long as the requirements are met.

Which of the following transactions would have to be reported on a Currency Transaction Report? A) An investor purchases $12,000 worth of securities with twelve $1,000 postal money orders. B) An investor purchases $40,000 worth of speculative and other low-grade bonds with a personal check. C) A 60-year-old customer directs his registered representative to liquidate $40,000 worth of growth mutual funds in his account and use the proceeds to purchase shares in a long-term bond fund. D) A new customer brings $40,000 worth of securities in to his broker-dealer firm for deposit into his account.

A Included as currency are cash, postal money orders, and traveler's checks.

You have a client who is about to retire and wants to rearrange his portfolio to have predictable income. Which of the following would not be a good investment vehicle? A) Income bonds B) AA-rated debenture C) AA-rated IDB D) U.S. Treasury note

A Income bonds, also known as adjustment bonds, are issued when a company is reorganizing and coming out of bankruptcy. Income bonds pay interest only if the company has enough income to meet the interest payment. As a result, these bonds normally trade flat without accrued interest. Therefore, they are not suitable for customers seeking income.

An investor in the 28% income tax bracket is considering purchasing either a 4% municipal bond or a 5% corporate bond. Which of the following statements regarding the two bonds' after-tax yields is true? A) The municipal bond's yield is higher than the corporate bond's yield. B) The yield difference cannot be determined. C) The corporate bond's yield is higher than the municipal bond's yield. D) The two bonds' yields are equivalent.

A Investors should invest in municipal bonds if the return after taxes is higher than comparable taxable bonds. To compare the two bonds, use the tax-free equivalent yield formula: (taxable yield) × (100% - tax bracket) = (tax-free equivalent yield). In this case, 5% × (100% - 28%) = 5% × 0.72 = 3.6%. Because the municipal bond yields 4% tax free, the investor should buy it; after taxes have been paid, the corporate bond yields only 3.6%.

When the inside market (best bid and best offer) for XYZ stock was 17.30-17.60, a market maker bought 100 shares from a customer at 16.90. At the time of the trade, the market maker's market was 17.25-17.70. What was the amount of the markdown? A) $0.40 B) $0.65 C) $0.75 D) $0.35

A Markdown is always based on the inside quote. In this case, the inside bid is 17.30 and the difference between that and the 16.90 buying price represents a $0.40 markdown.

A customer buys a newly issued municipal zero-coupon original issue discount bond for 85. If the bond is held until maturity, the tax consequence A) is $0. B) is $150 gain. C) is $150 loss. D) cannot be calculated from the information given.

A Municipal original issue discount bonds must be accreted. At maturity, the entire discount will be accreted, and the cost basis will be equal to the par value. No gain or loss will occur at maturity.

Which of the following mortgage-backed securities would provide investors with the most predictable maturity date? A) PACs B) Fannie Maes C) Ginnie Maes D) TACs

A PACs are planned amortization class CMOs and have established maturity dates. Prepayment risk is transferred to the PAC companion, or support, class bonds.

A corporate bond is quoted at 102⅝. A customer buying 10 bonds would pay A) $10,262.50. B) $10,285.00. C) $10,025.80. D) $10,258.00.

A Par ($1,000) × 102% = $1,020. Five-eighths of one bond point ($10) equals 0.625 times $10 equals $6.25. Therefore, the quote reading 102⅝ equals $1,026.25 per bond ($1,020 + $6.25). Because we are told the customer is buying 10 bonds, we multiply $1,026.25 by 10 bonds, which equals the amount the customer will need to pay to make the entire purchase: $10,262.50

A customer goes long 1 ABC June 40 call at 4 when buying the option in their account on Tuesday, April 4. When is the settlement date and what is the amount due? A) Wednesday, April 5; $400 B) Tuesday, April 4; $4,000 C) Thursday, April 6; $40 D) Wednesday, April 5; $4,000

A Regular way settlement on the stock trade when exercised would be T+2 business days.

Regulation T applies to A) both cash and margin accounts for nonexempt securities. B) margin accounts only for listed securities. C) margin accounts only for nonexempt securities. D) both cash and margin accounts for all unlisted securities.

A Regulation T controls the credit that broker-dealers extend in all types of accounts and only applies to nonexempt securities.

For each violation of FINRA or MSRB rules, FINRA may impose which of the following sanctions? Fines Censure Restitution Expulsion A) I, II, and IV B) I, III, and IV C) I and II D) I, II, III, and IV

A Restitution is not one of the sanctions that may be imposed by FINRA. However, that could be imposed by a court.

Funds for Life (FFL) is an SEC registered broker-dealer. The only securities business done by the firm is the sale of redeemable investment company securities. If FFL should go into bankruptcy proceedings, SIPC would A) not offer protection to any of the customers. B) offer protection up to $250,000 per customer. C) offer protection up to $500,000 per customer. D) protect any losses up to $250,000 in cash.

A SIPC, the Securities Investor Protection Corporation, is a nonprofit membership organization. SIPC members pay assessments into a general insurance fund that is used to meet customer claims in the event of a broker-dealer's bankruptcy

Your client owns 100 shares of CCC at $25. CCC declares a 25% stock dividend. After the ex-date, what will she own? A) 125 shares at $20 B) 100 shares at $31.25 C) 100 shares at $25 D) 125 shares at $18.75

A Stock dividends make the number of shares owned increase and the cost per share decrease. The overall value should remain unchanged before and after the adjustment: 125 shares × $20 = $2,500, and 100 shares × $25 = $2,500.

hich of the following statements regarding the suitability of municipal bonds are true? The tax-free interest payments make them more suitable for those in higher tax brackets. The tax-free interest payments make them more suitable for those in lower tax brackets. The tax-free interest is why municipal bonds are not considered suitable investments to be included in one's retirement account, such as an IRA. The tax-free interest is one reason why municipal bonds are considered suitable investments to be included in one's retirement account, such as an IRA. A) I and III B) I and IV C) II and IV D) II and III

A Tax-free interest payments are more suitable for those for whom the tax advantage has the most impact. That would be those in higher tax brackets, who would pay more taxes on the interest received if the interest payments were taxable. Additionally, the tax-free interest is why municipal bonds are not suitable for retirement accounts. This is because the earnings in retirement accounts are already tax deferred, and the impact of receiving tax-free interest is lost or diminished.

A direct participation program shows the following operations results: Revenues: $3 million Operating expense: $1 million Interest expense: $200,000 Management fees: $200,000 Depreciation: $3 million The profit or loss for the year is A) a loss of $1.4 million. B) a profit of $2.7 million. C) a loss of $3 million. D) a profit of $1.6 million.

A Taxable income for a partnership is determined as follows: Gross revenue: $3 million less operating expense: $1.2 million equals net revenue: $1.8 million less interest: $200,000 less depreciation: $3 million equals taxable loss: $1.4 million

Which of the following securities is frequently offered with a 50-year maturity? A) TVA bonds B) GNMA pass-through C) U.S. Treasury bond D) Federal Home Loan Bank

A Tennessee Valley Authority bonds are the only government security available today with a maturity as long as 50 years. Most of the agencies don't offer anything longer than 20 years, and the maximum on Treasury bonds is usually 30 years.

Which of the following statements regarding the Bond Buyer 20 bond index are true? It includes only GO bonds. It includes both GO bonds and revenue bonds. It is computed weekly. It is computed monthly. A) I and III B) II and IV C) I and IV D) II and III

A The Bond Buyer 20 bond index measures secondary market yields of GO bonds. It consists of 20 GO bonds, A-rated or better, and each with approximately 20 years to maturity. The index is updated each week.

Rank the following from first to last in order of payment at liquidation of a corporation. General creditors Preferred stock Subordinated debentures Accrued taxes A) IV, I, III, II B) III, IV, II, I C) IV, III, I, II D) III, IV, I, II

A The complete order of liquidation is as follows: secured debt, debentures and general creditors, subordinated debentures, preferred stock, common stock.

A convertible debenture has a conversion price of $40 per share. If the market value of the debenture rises to a 12.5-point premium over par, which of the following are true? Conversion ratio is 25:1 Conversion ratio is 28:1 Parity price of the common stock is $42 Parity price of the common stock is $45 A) I and IV B) II and III C) I and III D) II and IV

A The conversion ratio is computed by dividing par value by the conversion price ($1,000 par ÷ $40 = 25). The next step is calculating the market price of the debenture. A 12.5% premium to par means the market price is 112.5% of the $1,000 par. That computes to $1,125.00. Parity price of the common stock is computed by dividing the market price of the convertible debenture by the conversion ratio ($1,125 ÷ 25 = $45). Alternatively, if the debenture is at a 12.5% premium, the common stock will be at parity (equal) when it is selling at 112.5% of the $40 conversion price. Or, 112.5% × $40 = $45.

If an investor in the 27% federal marginal income tax bracket invests in municipal general obligation public purpose bonds nominally yielding 4.5%, what is the tax-equivalent yield? A) 6.16% B) 5.72% C) 16.67% D) 3.29%

A The formula for computing tax-equivalent yield is: nominal yield ÷ (1 − federal marginal income tax rate). Let's plug in the numbers: 0.045 ÷ (1 − 0.27) = 0.045 ÷ 0.73 = 6.16%. This tells the investor that they would have to receive 6.16% interest on a taxable bond to have the same after-tax return as earning 4.5% tax-free. You can check it out by working backwards. If an investor receives 6.16% taxable, they will have to pay 27% (in this person's bracket) in income tax. That is a tax of 1.66% (6.16 x 27% = 1.66). Subtract 1.66 in tax from 6.16 and the result is 4.50%.

Tenants In Common (TIC)

A form of joint ownership of an account whereby a deceased tenant's fractional interest in the account is retained by his estate. Ownership CANNOT be equal. Each party must specify a percentage interest in the account

An investor's portfolio contains the following four bonds: ABC 7% due in 2040 DEF 6% due in 2040 GHI 5% due in 2040 JKL 8% due in2040 Which of these bonds would show the greatest price change as a percentage of its current market price if interest rates jumped by one percentage point? A) GHI 5% due in 2040 B) ABC 7% due in 2040 C) DEF 6% due in 2040 D) JKL 8% due in 2040

A The longer the duration, the greater the price volatility (price change as a percentage of current value) when there is a change to market interest rates. When all bonds have the same (or approximately the same) length of time to maturity, the bond with the lowest coupon rate will have the longest duration. Conversely, the one with the highest coupon rate will have the shortest duration and changes in interest rates will have the least impact on it.

A customer buys 100 shares of HEX at 52, and at the same time, sells a HEX call for a premium of 4. What is his margin call deposit? A) $2,200 B) $1,560 C) $2,600 D) $1,300

A The margin call for the purchase of the stock is $2,600, and this is reduced by the sale of the call ($400) for a net of $2,200.

An investor in the 28% tax bracket has a $5,000 loss after netting all capital gains and losses realized. How much may the investor deduct from income that year? A) $3,000 B) $5,000 C) $2,500 D) $0

A The maximum deduction of net capital losses against other income in any one year is $3,000; any remaining loss can be carried forward into the next year.

If an investor buys one KLP Oct 95 put at 6.50, what is the investor's maximum potential gain? A) $8,850 B) $10,150 C) $9,650 D) $9,500

A The maximum gain on a long put is calculated by subtracting the premium from the strike price (95 − 6.50 = 88.50 per share). One contract represents 100 shares, so the buyer's maximum gain is $8,850 if the stock declines to zero. Because put buyers are bearish, they will make money if the stock falls below the breakeven point of 88.50.

An investor is long 500 shares of DEFG common stock, short 200 shares of DEFG common stock, and short 300 shares of DEFG 5% preferred stock. A tender offer for DEFG common shares is announced. Under SEC rules, this investor is permitted to tender A) 300 shares. B) 800 shares. C) 0 shares. D) 500 shares.

A The rules permit tendering of net long shares. That means the difference between the long and short positions. In this question, the investor is net long 300 shares: the difference between the 500 long and the 200 short. The preferred shares are not relevant because the tender is only for the common stock.

An investor purchased 200 shares of Hightown National Bank (HNB) common stock at $120.06 per share. Thirteen months later, HNB pays a 15% stock dividend. Three months after that, the investor sells the shares received from the stock dividend at $112.57 per share. The tax consequence to the investor is A) $245.10 long-term capital gain. B) $245.10 short-term capital gain. C) $122.55 short-term capital gain. D) $1,879.10 long-term capital gain.

A The total value of the initial position is unchanged, remaining at $24,012 (200 times $120.06). After the stock dividend the investor owns 230 shares (200 times 15% = 30 + 200 = 230). Therefore, the adjusted cost basis is $104.40 per share ($24,012 divided by 230 = $104.40). The question tells us that the investor sells those 30 additional shares at $112.57 per share. That is a difference of $8.17 per share. Multiply that gain by 30 shares and the result is a profit of $245.10.

A married couple has had an account with your FINRA member firm for many years. The account is registered in both names, JTWROS. Upon the advice of their estate-planning attorney, they wish to move the assets in equal proportion to individual accounts. This would require all of the following except A) a statement from the couple's attorney explaining the reason for the change. B) the essential facts relied upon by the person approving the change must be documented in writing and preserved with the customer account records. C) authorization of the change by a qualified registered principal designated by the member. D) before obtaining approval of the account designation change, a designated principal must be personally informed of the essential facts relative to the change.

A There is no FINRA requirement to receive any information from the couple's lawyer. All the other statements are correct.

An investor opens the following positions: Sell short 100 shares of BAF @61; short 1 BAF Sep 60 put @3¼. What is the customer's maximum gain, maximum loss, and breakeven point? A) Maximum gain is $425; maximum loss is unlimited; breakeven point is $64.25. B) Maximum gain is $5,775; maximum loss is $425; breakeven point is $64.25. C) Maximum gain is $325; maximum loss is unlimited; breakeven point is $57.75. D) Maximum gain is $425; maximum loss is $5,775; breakeven point is $57.75.

A This is a short sale of stock and a sale of a put option. The sale of the put provides some income and offers protection only to the extent of the premium. Short sellers want the stock's price to decline. They lose when it rises. The investor has received $6,425 ($6,100 from the sale of the stock and $325 from the sale of the option). That makes the breakeven point $64.25 per share. Once the price of the BAF stock goes above that, the investor loses money. Because there is no limit as to how high the stock's price can go, the maximum loss is unlimited. If, on the other hand, the stock's price declines into the 50s or lower, the owner of the 60 put will exercise and our investor will pay $6,000 to purchase the stock. That stock will be used to cover the short sale. That means the investor sold the stock (short) at $61 and bought it back at $60 for a gain of $100. At that point, the investor's profit is the $325 from the premium on the sale of the put plus the $100 gain (the difference between 61 and 60). That is why the maximum gain is $425

An investor purchases a zero coupon bond at a price of 64. The bond matures in nine years. Five years later, the investor sells the bond at a price of 80. This would result in A) a long-term capital loss of $40. B) a long-term capital loss of $200. C) a long-term capital gain of $160. D) no gain and no loss.

A This question deals with accretion of the discount. The discount here is $360 (the difference between the $640 paid and the $1,000 maturity value). With nine years until maturity, the annual accretion is $360 divided by nine, or $40 per year. After five years, the bond's basis has increased by $200 ($40 times 5 years) to $840. The sale at $800 represents a long-term loss of $40.

A new customer is opening a cash account with your broker-dealer. The new account form must contain the signature(s) of A) the principal only. B) the principal and the registered representative working with the customer. C) the customer only. D) the customer and the registered representative working with the customer.

A To open a cash account, only the signature of the principal accepting the account is required. For margin accounts, the signature of the customer would be required on the margin agreement.

Your client has purchased shares of VACL at several different times. A view of the client's account ledger indicates the following: 100 shares @$50 on February 12 100 shares @$52 on April 23 200 shares @$49 on May 12 100 shares @$55 on June 28 The client decides to sell 200 shares of the VACL on November 14 of the same year when the price of the stock is $53 per share. Absent any instructions to the contrary, for tax purposes, the client will report a short-term capital A) gain of $400. B) loss of $200. C) gain of $300. D) gain of $800.

A Unless the client gives specific instructions, the IRS will always use the FIFO method of determining the shares sold. In this case, it would be the 100 purchased in February at $50 and the 100 purchased in April at $52. That is a $300 profit on the February purchase and a $100 profit on the April one. That totals $400. Because all transactions are in the same year, any gain (or loss) is short term.

The owner of an IRA, age 45, has contributed $10,000 into the account and the IRA is now worth $20,000. The owner is going to convert the entire $20,000 into a Roth IRA. What are the tax consequences of this conversion? A) The $20,000 is taxable as ordinary income in the year of the conversion. B) The $20,000 is taxable as ordinary income, but there is a $2,000 tax penalty for early withdrawal. C) $10,000 will be taxable as ordinary income, and $10,000 will be taxed as a capital gain. D) $10,000 will be taxable as ordinary income, and $10,000 will be taxed as a capital gain; in addition, there will be a $2,000 tax penalty for early withdrawal.

A When converting from a traditional IRA to a Roth IRA, the distribution is all taxed as ordinary income in the year of the conversion. There is no 10% tax penalty if the conversion is done prior to age 59½.

A company has filed for an IPO at $20 par value. The IPO is priced at $30 per share. Where on the balance sheet is the extra $10 recorded? A) Capital surplus B) Retained earnings C) Distributed dividends D) Excess par value

A When new stock is issued, any funds paid for the stock in excess of the par or stated value is called capital surplus. In this case, it is the $10 above the $20 par. It might also appear on your exam as paid-in surplus.

With ABC trading at 39, a customer buys 1 ABC Mar 40 call and sells 1 ABC Mar 35 call. A profit occurs if the spread widens. the spread narrows. ABC declines sharply. both contracts are exercised. A) II and III B) II and IV C) I and III D) III and IV

A Whenever there is a question where the choices are widen or narrow, you first need to determine if the spread is a debit or a credit. A debit spread is one where the option you buy costs more than what you receive for the one you sell. A credit spread is the opposite—the one you are selling has a higher price than the one you are buying. In this question, the premiums are not given, so we have to figure out which is the more expensive option. We have a 35 call and a 40 call. Which of those is more valuable? The ability to buy the stock at 35 or buy it at 40? The lower the exercise (strike) price of a call, the greater the value, so we know the premium on the 35 call will be higher than that of the 40 call. Therefore, selling the 35 and buying the 40 is going to be a credit spread because more money will come from the sale of the 35 that will go out for the purchase of the 40. Once we know it is a credit spread, we go to the answer choice narrow. And, when we want the spread to narrow, we want the options to expire, so IV cannot be correct.

An investor in a high-income tax bracket owns a number of municipal bonds and wants to add some to a 401(k) plan he participates in and perhaps his IRA. As a registered representative, you would advise that this is A) not suitable because the interest payments from municipal bonds are tax free already and have no place in a tax-advantaged (tax-deferred) account such as a 401(k) plan or IRA. B) not suitable because the investor already owns municipal bonds, and this would be a duplication of the same asset class in his tax-advantaged accounts. C) suitable because the investor already understands the advantages of owning tax-free interest paying instruments without any further suitability qualifications needed. D) suitable due to his high-income tax bracket.

A While municipal bonds can be suitable for those in higher income tax brackets, they have no place in tax-advantaged (tax-deferred) accounts such as 401(k) plans or IRAs because the interest paid is already tax free.

A bond has a 7% coupon and an offering price of 108. The bond matures in ten years. An investor purchasing this bond at the offering price would have a yield to maturity closest to A) 5.96% B) 7.50%. C) 7.22%. D) 7.80%.

A With a coupon rate of 7%, the answer must be something less than that. "When you pay more, you get less,"

Based on yesterday's closing price of $60 per share, Blech Sheet Metal, Inc., has a current P/E ratio of 12:1. If the current quarterly dividend payment is $0.50 per share, the dividend payout ratio is A) 40%. B) 10%. C) 3.33%. D) 16.66%.

A With a price-to-earnings ratio of 12:1, the earnings per share (EPS) is $5.00 ($60 divided by 12). Four quarterly dividends of $0.50 is an annual dividend of $2.00 per share. If the company is paying $2 per share from the $5 per share earnings, that is a dividend payout ratio of 40%.

Which of the following statements regarding qualified retirement plans are true? Contributions are made with pretax dollars. Contributions are made with after-tax dollars. Distributions are 100% taxable. Distributions are taxable only to the extent of earnings. A) I and III B) I and IV C) II and III D) II and IV

A With qualified plans, participants receive a tax deduction for contributions to their plan. As earnings accumulate tax deferred, distributions, which consist of tax-deferred earnings and contributions for which the participant received a tax deduction, are 100% taxable.

he Interstate Bridge Authority has an outstanding revenue bond. For the most recent operating period, the Authority has net revenue of $36 million, operations and maintenance expenses of $16 million, debt service requirements of $18 million, and surplus funds of $2 million. The debt service coverage ratio for the Interstate Bridge Authority's revenue bond is A) 2:1. B) 1:1. C) 2.25:1. D) 1:11:1.

A he debt service coverage ratio is determined by dividing the net revenue by the debt service requirement. In this question, the debt service coverage ratio would be 2:1 ($36 million divided by $18 million = 2). If you subtracted the $16 million of expenses because you did not notice that we gave you the net revenue, your ratio was 20 divided by 18 = 1.11 to 1. If you added the surplus (not an expense), your ratio was 18 divided by 18 = 1:1. It is not uncommon to have information in a question that is not needed to arrive at the solution

A customer purchased 10 ABC 9s of 2045 convertible debentures at 99. The debentures are callable at 101. The conversion ratio is 40. Some time later, the debentures are called while the common is trading at $24 and the debenture is trading at 98. Which of the following options would be most beneficial to the customer? A) Tender the bonds to the corporation B) Sell the bonds C) Wait for a better offer from the corporation D) Convert the bonds and sell the common stock

A recognize that the investor purchased 10 of the debentures. They have a coupon of 9% and mature in 2045. None of that is relevant to answering the question, but we want to be sure you understand the terminology.The option most beneficial to the investor is tendering the debentures to the corporation for $10,100 (10 times $1,010). If the debentures were sold on the market, the investor would receive $9,800 (10 times $980). If the debentures were converted into common, the investor would receive 400 common shares (40 shares per debenture times 10) that could be sold for their current price of $24, for a total of $9,600.

If a 50% stock dividend is declared, the owner of 1 XYZ Jul 30 call owns A) one contract for 150 shares with an exercise price of 20. B) two contracts for 100 shares with an exercise price of 20. C) two contracts for 150 shares with an exercise price of 30. D) one contract for 100 shares with an exercise price of 20.

A (100 × 150% = 150 shares). The effective strike price is adjusted so that the position value remains the same before and after the adjustment—in this case, 20 ($3,000 / 150 shares = $20).

The Jefferson County Water Works revenue bond is being underwritten by a syndicate led by ABC Securities, Inc. The bond has serial maturities going out up to 25 years with a balloon at 30. The coupons range from 3.2% to 4.1%, and all the bonds are offered at par. The terms of the syndicate agreement call for a total takedown of ¾ of a point with a selling concession of ½ point. A syndicate member who sells 500 of the bonds will earn A) $3,750. B) $7,500. C) $6,250. D) $2,500.

A The computation is 500 bonds sold × $7.50 per bond = $3,750 underwriting profit.

If a customer with no other position sells 1 KLP Jul 80 call for 10 and buys 100 shares of KLP stock for $85 per share, he will break even when KLP stock is trading at A) $75. B) $95. C) $92. D) $70.

A - Breakeven for a covered call writer is the purchase price less premiums received. In this case, breakeven is $85 minus $10, or $75 per share; below $75, the customer loses money.

Regulation T requires payment from a customer in a margin account A) within four business days. B) before purchase. C) business 10 days. D) within three business days.

A - The Regulation T payment date is the fourth business day from the trade date. Regular way settlement, according to FINRA rules, is two business days from the trade date, and Regulation T allows for two additional business days after settlement date—four business days total.

Which type of individual account allows for investments held in that account to go straight to a named beneficiary outside of probate? A. A TOD account B. A testamentary account C. An account titled JTWROS D. An advisory account

A. A simple way for an individual account owner to ensure that the assets in the account pass directly to the named beneficiary is to use the TOD option.

An IRA account at a broker-dealer must be set up as A. a cash account B. a margin account C. a wrap account D. an advisory account

A. IRAs can only be opened in a cash account

All of the following are true regarding community property except A. Property acquiried before marriage is considered to be jointly owned B. Community property is considered to be owned jointly by both spouses and would be divided at the time of divorce, annulment, or death. C. Not all states are community property states D. An exception to community property is for property that was inherited

A. Property acquired before marriage is considered to be owned separately, not jointly, as stated in the answer choice. Each of the choices is a true statement

Sole Properietorship

All income (or loss) is that of the individual. All of the owners assets are liable for the debts of the business.

A collateralized mortgage obligation (CMO) makes an interest-only payment to an investor. This payment will be A) tax free. B) taxed as ordinary income. C) taxed as a capital gain if the underlying mortgage is prepaid. D) treated partly as ordinary income and partly as a tax-free return of principal.

B

A customer asks for sales literature for a money market mutual fund. Upon receiving the literature, she notices A) the fund's current yield is insured by the Federal Deposit Insurance Corporation. B) the fund seeks to maintain a stable price but the fund can lose money. C) the fund stipulates that future results will be substantially the same as the past performance listed in the 1-, 5-, and 10-year returns. D) the fund is an appropriate addition for investors seeking capital appreciation.

B

A customer is very concerned about investments that may not keep pace with inflation. He asks which securities would have the least exposure to inflation risk. Which of the following would be the best answer? A) Cash B) Common stock C) Fixed annuity D) Preferred stock

B

A document that allows an investor in Class A shares of a mutual fund to receive a breakpoint on an initial purchase without investing the required breakpoint amount is A) the new account form. B) a letter of intent. C) the rights of accumulation form. D) the breakpoint sale memorandum.

B

A generic ad for an investment company placed by a broker-dealer would contain A) both the name of the investment company and the name of the broker-dealer. B) the name of the broker-dealer, but not the name of the investment company. C) neither the name of the investment company nor the name of the broker-dealer. D) the name of the investment company, but not the name of the broker-dealer.

B

A limited partner (LP) invests $100,000 in a movie production limited partnership with a nonrecourse note for $300,000. The partnership liquidates, and the LP receives $100,000. His loss, for tax purposes, is A) $200,000. B) $0. C) $100,000. D) $300,000.

B

A married couple has several individual and joint accounts with your firm. One spouse calls you and requests that you make a transfer of funds between the accounts. This would not present a problem if A) the caller is a signatory on the account receiving the funds. B) the caller is a signatory on both accounts. C) you verify the identity of the caller. D) you have the caller send the request in writing.

B

A member of the board of directors of the Able Baker Charlie Company (ABCC) took her director's fees and purchased 200 shares of ABCC on the Nasdaq Stock Market at $20 per share. If she wished to sell these shares, compliance with Rule 144 would entail A) meeting both a size limit and a time limit. B) meeting neither a size limit nor a time limit. C) meeting a time limit, but not a size limit. D) meeting a size limit, but not a time limit.

B

A municipal bond dealer is making a bona fide quote. Which of the following statements regarding such a quote is true? A) The quote cannot represent an offer to sell bonds that the dealer does not currently own. B) The quote must have a reasonable relationship to fair market value. C) The quote need not be one that the dealer is prepared to act upon (buy or sell). D) The quote may not take into consideration any anticipated market movement.

B

A properly signed and guaranteed stock or bond power is A) a legal right of the owner or proxy to vote stock as he chooses. B) a document that can be attached to a certificate, authorizing transfer of ownership to another party or when a security is hypothecated. C) an authorization delegating voting power to another person. D) a legal power of attorney authorizing the bulk transfer of variable annuities, whose separate accounts are invested in more than 75% equity or debt, to a new custodian.`

B

A purchase or redemption order for investment company shares must be executed at a price based on A) the best net asset value computed the same day the fund receives the order. B) the net asset value next computed after the fund receives the order. C) the net asset value last computed before the fund receives the order. D) the net asset value computed at the close of trading on the NYSE the day before the fund receives the order.

B

A registered representative must obtain written verification of an investor's net worth for which of the following investments? A) Mutual fund B) Direct participation program C) Variable contract D) Real estate investment trust

B

A registered representative of a FINRA member firm has been found guilty of a trade practice violation. If desired, the Code of Procedure requires the individual to file an appeal within A) 30 days after receiving the decision. B) 25 days after receiving the decision. C) 60 days after receiving the decision. D) 45 days after receiving the decision.

B

A transaction made for regular way settlement occurs on Thursday. The following Monday is President's Day, when the stock markets are closed. The buying broker-dealer is domestic, while the selling broker-dealer is foreign based. The trade took place on the Nasdaq Stock Market. Settlement between the broker-dealers would take place A) on Friday, the business day preceding the holiday. B) on Tuesday, the next business day following the holiday. C) on Monday, because the seller is foreign based. D) on the date specified as a seller's option.

B

All of the following are characteristics of a direct participation program (DPP) except A) its revenue, deductions, and tax credits are reported to the IRS. B) it pays federal income taxes. C) its revenue, deductions, and tax credits are proportionally given to investors. D) the most common structure used for a direct participation program is the limited partnership.

B

All of the following are reasons to consider adding REITs to your client's portfolio except A) low or negative correlation to the stock market. B) redemption at net asset value. C) reasonable income from mortgage REITs. D) capital appreciation from equity REITs.

B

All of the following must be considered by an investment adviser representative before recommending a municipal general obligation (GO) bond to a customer except A) the customer's tax status. B) the municipality's coverage ratio. C) the municipal security's rating. D) the customer's state of residence.

B

All of the following securities are exempt from the registration provisions of the Securities Act of 1933 except A) state and municipal bonds. B) commercial bank holding company securities. C) commercial paper and bankers' acceptances that have maturities of no more than 270 days. D) national and state bank securities.

B

All of the following statements regarding collateralized mortgage obligations (CMOs) are true except A) principal repayments are applied to earlier tranches first. B) interest is paid semiannually. C) interest payments are distributed pro rata when received. D) CMOs are a derivative security.

B

All of the following would be considered a proper way for a client to file a complaint with a FINRA member firm except A) text. B) telephone. C) instant message. D) telegram.

B

An investment banker purchasing what is left unsold from a rights offering is engaging in A) preemptive rights underwriting. B) standby underwriting. C) all or none underwriting. D) firm commitment underwriting.

B

An investor might expect to receive the greatest gain on an investment in a corporate bond by purchasing A) short-term bonds when interest rates are high. B) long-term bonds when interest rates are high. C) long-term bonds when interest rates are low. D) short-term bonds when interest rates are low.

B

An investor purchased a municipal bond at par to yield 5.5% to maturity. If, two years later, she sold the bond at a price equivalent to a 5% yield to maturity, the investor incurred A) no taxable result at this time. B) a capital gain. C) a capital loss. D) taxable interest income.

B

An investor purchased an interest in a limited partnership, paying $10,000 in cash and signing a recourse note to the partnership under a letter of credit for $40,000. Which of the following statements are true? The investor's tax basis will be $10,000. The investor's tax basis will be $50,000. The investor's maximum loss will be $10,000. The investor's maximum loss will be $50,000. A) II and III B) II and IV C) I and IV D) I and III

B

Bob Smith, who is in his 40s, has just become covered by an extremely generous defined benefit retirement plan at his company. He has decided he no longer needs his variable annuity for retirement purposes and wants to use the money for a trip to Africa. Over the past 10 years, he has invested $60,000 in the annuity, and its net value is now $80,000. If Bob should go ahead and surrender the annuity, the tax consequences will be A) capital gains tax on $20,000 and a $2,000 penalty. B) ordinary income tax on $20,000 and a $2,000 penalty. C) capital gains tax on $60,000 and a $6,000 penalty. D) ordinary income tax on $60,000 and a $6,000 penalty.

B

Common risk factors found when investing in penny stocks would include all of the following except A) high volatility. B) unfavorable tax treatment. C) illiquidity. D) limited transparency.

B

FINRA Rule 2330 states: no member or person associated with a member shall recommend to any customer the exchange of a deferred variable annuity unless such member or person associated with a member has a reasonable basis to believe the exchange suitable, taking into consideration whether A) the new annuity has a higher assumed interest rate. B) the customer has had another deferred variable annuity exchange within the preceding 36 months. C) the customer's health has declined since the purchase of the initial annuity. D) the customer is at least 59½ and will not be subject to the 10% tax penalty.

B

If a customer of your firm receives stock from the estate of her mother, the stock's cost basis in the hands of the customer is A) the market value at date of distribution to the customer. B) the market value at date of death. C) the original cost of the stock. D) the original cost of the stock adjusted for any estate taxes paid.

B

Index options differ from equity options in which of the following ways? A) They are subject to closing purchases as well as closing sales. B) The exercise settlement is in cash. C) The trade settlement date is the next business day. D) The expiration date is the third Friday of the expiration month.

B

Maintaining a fair and orderly market on the NYSE trading floor is the responsibility of A) the order book official. B) the designated market maker. C) the traders. D) the floor brokers.

B

One of your individual customers would like to add some foreign debt securities to their portfolio. When told that the investment would be $2,500, the best suggestion would be to A) tell the customer that $2,500 is below the minimum purchase quantity of foreign bonds. B) invest in a mutual fund concentrating in foreign debt securities. C) use one of the overseas branches of your firm to suggest the appropriate issues. D) contact a broker-dealer in the foreign country of choice and open an account there.

B

Regarding a summary section and a statement of additional information (SAI) for management investment companies, which of the following is true? A) Both must be included in the prospectus of a management company. B) A statement of additional information need not be included in the prospectus of a management company. C) A summary section need not be included in the prospectus of a mutual fund. D) Neither are required to be in the prospectus of a mutual fund.

B

The 5% markup policy would apply to all of the following equity transactions except A) a proceeds transaction. B) a primary market transaction. C) an agency trade done on an exchange. D) a riskless principal transaction.

B

The call premium on a municipal bond trading above par is best described as the difference between A) the market price and par. B) par and the call price. C) the amortized premium and the annual interest. D) the market price and the call price.

B

The price of which of the following will fluctuate most with a change in interest rates? A) Common stock B) Long-term bonds C) Short-term bonds D) Money market instruments

B

The writer of an equity call option who is assigned A) can enter a closing transaction any time before exercise settlement. B) must deliver stock within two business days. C) must deliver stock within one business day. D) can enter a closing transaction on the day the exercise notice is received.

B

There are different record retention requirements for different records. Under FINRA Rule 4513, how long must a FINRA member firm keep a record of written customer complaints? A. One year B. Four years C. Six years D. Lifetime of the customer's account

B

When discussing mutual funds with a customer, which of the following statements is not prohibited? A) Get a few friends to join with you to form an investment club, and you may qualify for a breakpoint. B) Buy shares of different funds in the same fund family, and you may qualify for a breakpoint on the total purchase. C) Buy the shares on record date to receive the dividend. D) The income yield of the fund consists of both dividends and capital gains.

B

When investing in securities, there are many potential risks. When recommending a specific security to clients, a member firm A) may follow the dictum of caveat emptor. B) must disclose the existence of a control relationship between the firm and the subject company. C) must have a reasonable belief that the recommended security will outperform the overall market. D) may disclose the existence of a control relationship between the firm and the subject company if it is material.

B

Which of the following forms of business is preferred when the goal is raising a significant amount of capital? A) General partnership B) C corporation C) S corporation D) LLC

B

Which of the following is an example of sovereign debt? A) Royal Bank of Canada CDs B) U.S. Treasury bonds C) Bank of England notes D) Sony Corporation debentures

B

Which of the following oral orders can be accepted from a customer without additional documentation? A) Increase my position in ABC B) Buy 100 shares of ABC when the price is right C) Buy $20,000 of quality bank stocks D) Buy 200 shares of computer stock

B

Which of the following regarding revenue bonds are true? They are secured by a specific pledge of property. They are a type of general obligation bond. They are not subject to the statutory debt limitations of the issuing jurisdiction. They are analyzed primarily on the project's ability to generate earnings. A) I and IV B) III and IV C) II and III D) I and II

B

Which of the following regarding yield-based (interest rate) debt options is true? A) Their strike prices reflect dollar amounts. B) They are European-style exercise. C) Debt securities are delivered to the contract owner when exercised. D) Calls are purchased by those who believe prices of debt securities are rising.

B

Which of the following statements regarding Treasury bills are true? They are sold in minimum denominations of $10,000. They are offered with maturities ranging up to 52 weeks. Their interest is exempt from taxation at the state level. They are callable by the U.S. Treasury at any time before maturity. A) II and IV B) II and III C) I and II D) I and III

B

Which of the following statements regarding the Code of Arbitration Procedure is true? A) Simplified arbitration is available for claims of $50,000 or less, and the statute of limitations is three years from the triggering event. B) Simplified arbitration is available for claims of $50,000 or less, and the statute of limitations is six years from the triggering event. C) Simplified arbitration is available for claims of $25,000 or less, and the statute of limitations is six years from the triggering event. D) Simplified arbitration is available for claims of $25,000 or less, and the statute of limitations is three years from the triggering event.

B

Which of the following statements regarding the flow of funds found within a municipal trust indenture are true? It describes the disbursement of funds for revenue bond issues. It describes the disbursement of funds for general obligation issues. It is found within the official statement. It is found within the bond contract. A) I and III B) I and IV C) II and IV D) II and III

B

Which of the following taxes are considered sources of debt service for special tax bonds? Ad valorem tax License taxes paid by businesses Special liquor and tobacco taxes Real estate taxes A) II and IV B) II and III C) III and IV D) I and III

B

Which of the following would not be a valid use of the partnership democracy? A) Consenting to a legal judgment against the partnership B) Deciding which partnership assets should be liquidated to pay creditors C) Removing the general partner D) Consenting to an action of a general partner that is contrary to the agreement of limited partnership

B

Which type of historical information is required on Form U4? I. 5-year residency II. 10-year residency III. 5-year employment IV. 10-year employment A. I and III B. I and IV C. II and III D. II and IV

B

With ABC stock selling for $49, a client sells one ABC 50 Nov call option in his cash account with your firm. One week later, ABC is now at $51 per share and his spouse sells two ABC 50 Nov calls in her account. In early November, ABC is selling for $62 per share and the spouse is assigned an exercise notice on one of the calls. The client calls and asks you, "Why was the exercise notice assigned to my spouse and not me?" You should respond: A) your broker-dealer assigns exercise notices based on the larger position. B) your broker-dealer uses random allocation when assigning exercise notices. C) your broker-dealer assigns exercise notices based on LIFO. D) your broker-dealer assigns exercise notices based on the market price at the time the option was written.

B

A couple's home has an assessed value of $40,000 and a market value of $100,000. What will the tax be if a rate of 5 mills is used? A) $2,000 B) $200 C) $500 D) $5,000

B ($40 × 5 mills = $200).

A customer establishes the following positions: Buy 100 ABC at 28 Buy 1 ABC Dec 25 put at 2 What is the breakeven point? A) 23 B) 30 C) 26 D) 27

B (28 + 2 = 30)

A J&J Treasury bond with a 5% coupon having a maturity date of July 1, 2039, is purchased in a transaction for cash on February 24. What is the number of days of accrued interest shown on the trade confirmation? A) 63 B) 54 C) 53 D) 55

B A bond begins accruing interest on the prior interest payment date (January 1) and accrues up to, but not including, the settlement date (February 24). Did you notice that this was a transaction for cash? That means the settlement date is the same day as the trade (February 24). Normally, Treasury securities settle T+1. If this was a regular-way trade, the accrued interest would be 55 days because settlement would have been February 25. Be careful reading the question; it is easy to skip over critical information. Because accrued interest on government bonds is computed actual days, actual year, 31 days for January plus 23 days for February, it equals 54 days.

If a customer attempts to place an order for municipal securities that the registered representative deems completely unsuitable for the customer, the registered representative A) must obtain the permission of a municipal securities principal before executing the order. B) may execute the order and mark the order ticket as unsolicited. C) must refuse to execute the order. D) may execute the order on a not held basis.

B A registered representative may enter any unsolicited order from a customer despite the fact that the representative believes the trade to be unsuitable for the customer. The representative must mark the order ticket unsolicited.

Which of the following statements regarding a unit investment trust is not true? A) It is considered an investment company. B) Overall responsibility for the fund rests with the board of directors. C) It invests according to stated objectives. D) It charges no management fee.

B A unit investment trust (UIT) has no board of directors; rather, it has a board of trustees

Variable annuities must be registered with the state banking commission. the state insurance commission. the Securities and Exchange Commission (SEC). FINRA. A) II and III B) II and IV C) III and IV D) I and III

B A variable annuity is a combination of two products: an insurance contract and a mutual fund. Therefore, variable annuities must be registered with the state insurance commission and the SEC.

One of the ways in which U.S. government agency issues differ from those offered directly by the U.S. Treasury is that A) agency issues are taxable on the federal level, while Treasury issues are not. B) agency issues typically carry higher returns than Treasury issues because of the lack of direct government backing. C) agency issues are more likely to be issued in larger amounts. D) agency issues frequently trade on the NYSE, while Treasuries never do.

B Agencies, with very few exceptions (GNMA being one), do not carry the direct backing of the U.S. Treasury. While they are quite safe, that lack of direct backing causes their yields to be somewhat higher. Agencies are never traded on the stock exchanges, and their float is almost always smaller than Treasuries. Both are taxable on the federal level.

Which of the following is not a right conferred upon ownership of common stock? A) Voting in person or by proxy B) Limited liability C) Transferability of shares D) Dividends, if declared by the board of directors

B Although ownership of common stock means the holder's maximum loss is limited to the original investment, it is not a stockholder right. The doctrine of limited liability is a legal construct and shields stockholders from being responsible for debts of the company. Being able to vote the shares; being able to sell them without needing the issuer's permission; and dividends, if declared, are considered rights of owning stock.

A father opens four custodial accounts for each of his children with the same mutual fund company. He invests $15,000 in each account. The fund company has breakpoints at $50,000, $100,000, and $200,000. The sales charge is A) based on a $15,000 investment and is levied for each account. B) based on the total $60,000 investment and qualifies for the $50,000 breakpoint. C) based on the total $60,000 investment and qualifies for the $100,000 breakpoint. D) ineligible for a breakpoint discount because these are custodial accounts.

B An investment made by one person in four custodial accounts for his children at the same fund company would qualify for the breakpoint that is applicable to the total amount invested. In this case, $60,000 was invested, so the applicable breakpoint is at $50,0

All of the following statements regarding a qualified pension plan are true except A) it must cover all of its eligible employees. B) growth in the account is tax free. C) it must comply with nondiscrimination rules. D) it requires advance approval from the IRS.

B Growth in qualified pension plans, as well as other qualified plans, is tax deferred, not tax free. All growth is taxable at the time of distribution.

A customer who owns TCB stock wants to continue holding the security. The stock has fallen from 26 when he bought it on February 2 to a 52-week low of 20.75. He sells the stock on December 1 at the low and repurchases it at 21 on December 15. What is the tax consequence of this investment? A) The holding period for the stock was wiped out. B) The tax loss is not allowed. C) He has a capital loss. D) By repurchasing the investment at the same price, he keeps the original cost basis.

B Because the security was repurchased in less than 30 days, the IRS will not allow the loss due to the wash sale rule. It would have been allowed had the customer bought back the security after 30 days.

KLM Company has 10 million convertible bonds outstanding that are convertible at $25. The bonds contain an antidilution feature. If KLM declares a 10% stock dividend, the new conversion price will be A) $50.00. B) $22.73. C) $22.50. D) $45.45.

B Before the stock dividend, an investor would have received 40 shares of stock for each $1,000 bond ($1,000 / $25). A 10% stock dividend would now give an investor 44 shares on conversion (40 shares + 10% = 4 shares more). $1,000 / 44 shares = $22.73 per share for the new conversion price.

An investor had a $20,000 capital loss, a $15,000 capital gain, and $50,000 in income for the year. How much of the income is taxable? A) $30,000 B) $47,000 C) $20,000 D) $50,000

B Capital losses may be used to reduce taxable income. The first step is to net the gains and the losses. This investor has a net loss of $5,000. Of that net loss, a maximum of $3,000 can be written off against the income for the year. That reduces the investor's taxable income to $47,000. The unused $2,000 of the net loss is carried forward to subsequent tax years until utilized.

Nickelplate Manufacturing Corporation (NMC) is capitalized with 1 million shares of a 6% $50 par callable preferred stock and 10 million shares of $1 par common stock. With the preferred stock currently selling at $75 per share and the common stock at $60 per share, the current yield of the preferred stock is closest to A) 8%. B) 4%. C) 5%. D) 6%.

B Current yield on any security, stock or bond, is the annual income (dividend on stock, interest on bond) divided by the current market price per share (or per bond). The math in this question is the dividend of $3 (a 6% $50 par preferred stock is paying an annual dividend of 6% of $50, or $3 per share) divided by the current market price of the preferred stock ($75). The quotient is .04 or 4%. What about the common stock? All of that information is just to distract you. We cannot compute the current yield of the common stock because we do not have any information about its dividend.

ABC Company has issued $20,000,000 of convertible bonds with a coupon of 5% and a current market value of 120. The conversion price is $40. If all the bonds are converted, how many additional shares of common stock will ABC have outstanding? A) 400,000 B) 500,000 C) 1,000,000 D) 600,000

B Each bond will convert to 25 shares of common stock ($1,000 ÷ $40). 20,000 bonds were issued ($20,000,000 ÷ $1,000). Therefore, 500,000 additional shares (20,000 × 25) will be outstanding if all the bonds are converted.

One of your customers notices that the short interest on the NYSE is high. When she asks you for an interpretation, you should tell her that this signals A) a period of stability in the market. B) a bullish market. C) a period of volatility in the market. D) a bearish market.

B Even though short interest represents the number of shares sold short, many investors consider it a bullish indicator when this number is high. Each share that has been sold short must be replaced (covered) at some point.

All of the following statements regarding the risks of investing in an oil and gas limited partnership are true except A) development programs may involve acquisition of expensive leases. B) development programs have higher risk than exploratory programs. C) wells may not have sufficient reserves to return drilling costs. D) income programs have fewer tax benefits than exploratory programs.

B Exploratory programs have the highest risks, rewards, and tax benefits. Development wells are drilled to develop a reserve that is already known to be present.

When an account is owned by an individual who is 65 years old or older, or the client is 18 years old or older and a member firm believes the client has an impairment that prevents the person from defending their interests, a temporary hold is permitted on disbursements for how many days if the firm comes to reasonably believe that an attempt at exploiting the person has been made? A) 5 business days B) 55 business days C) 3 business days D) 30 calendar days

B FINRA Rule 2165 permits firms to place a temporary hold of 55 business days on disbursements from the accounts of individuals aged 65 or older and individuals aged 18 or older whom firms reasonably believe have an impairment that prevents them from protecting their own interests (a specified adult). This is done if the firm has a reasonable belief that financial exploitation of the specified adult has occurred or been attempted. This gives the firm time to notify a trusted contact person and begin an internal review.

Your 30-year-old client has $100,000 to invest and is willing to assume a moderate amount of risk, but she would also like to have $10,000 available for a down payment on a home in six months. Which of the following asset allocation strategies would best suit her situation? A) 70% high-yield corporate bond fund, 20% growth fund, 10% government bond fund B) 70% large-cap stock fund, 20% balanced fund, 10% money market fund C) 50% large-cap stock fund, 40% municipal bond fund, 10% money market fund D) 50% government bond fund, 50% large-cap fund

B First we have the short term of six months for the home down payment, so she'll need capital preservation and liquidity. That is accomplished with the money market fund. Then, being 30 years old, she has a long-term time horizon that necessitates investing for growth and inflation protection. That is where the 70% in large-cap securities is the most appropriate asset allocation for her. The 20% in the balanced fund helps keep the overall risk level on the moderate side. One point to remember is that municipal bonds (or municipal bond funds) will never be the correct investment choice unless the question states that the client is in a high tax bracket or is looking for tax-free income.

Which of the following would protect a short May 50 put? A) Long Jun 45 put B) Long Jun 55 put C) Long Apr 55 put D) Long Apr 45 put

B For a long put to cover a short put, it must have the same or higher strike price and the same or longer expiration. This ensures the investor may sell the stock without financial loss if the short put is exercised, and she is forced to buy.

Pursuant to Regulation T, cash dividends received in a customer's margin account A) must be removed within 30 days of receipt. B) can be withdrawn anytime within the first 30 days of receipt. C) can only be withdrawn if the account is not restricted. D) cannot be removed.

B If a customer wishes to withdraw cash dividends, the customer must do so within 30 days of receipt. Otherwise, they become a permanent reduction of the debit balance. The customer does not lose the dividend; rather, the dividend amount is now reflected as increased equity in the account. As the debit balance falls, equity in the account goes up dollar for dollar.

A customer gives you a limit order to buy 500 shares of XYZ at 30. You erroneously buy 1,000 shares at 29. The customer is entitled to A) 1,000 shares at 30. B) 500 shares at 29. C) 500 shares at 30. D) 1,000 shares at 29.

B If the firm buys more shares than indicated on the order ticket, the customer cannot be held responsible.

A member of a $5 million Eastern account that has a $500,000 participation fails to sell $200,000 of bonds. At the close of the offering, if $1 million worth of bonds remains unsold, the member must take down A) $500,000. B) $100,000. C) $300,000. D) $200,000.

B If the member was liable for 10% of the issue's original dollar value, it is committed to take down 10% of any bonds remaining unsold (10% of $1 million equals $100,000).

In early September, a customer buys 100 shares of MCS stock for $83 per share and simultaneously writes 1 MCS Mar 90 call for $4 per share. The customer will break even when MCS stock is at A) $86. B) $79. C) $94. D) $87.

B If the stock rises above $90, the writer will be exercised and will make $700 on the stock (buy at $83, deliver at $90) and keep the $400 received in premiums. If the stock declines, the call expires unexercised. The writer can lose $400 on the stock (the premiums earned) and still break even. This occurs at $79 ($83 − $4). Breakeven is the cost of stock purchased minus premiums.

If a customer sold 1,000 shares of XYZ at a loss, a wash sale will result within 30 days of the date of sale if your customer A) writes 10 XYZ at-the-money puts. B) buys 10 XYZ at-the-money calls. C) buys 10 XYZ at-the-money puts. D) writes 10 XYZ at-the-money calls.

B If, within 30 days of the date of sale, the customer buys back the security or the right to buy it back (a call option), the loss is disallowed. It will also be disallowed if the customer writes deep in-the-money puts on the security sold within 30 days. A deep in-the-money put will likely be exercised, forcing the customer to buy stock.

In an account opened by two individuals as joint tenants with right of survivorship, all of the following are true except A) mail may be directed to the joint owner agreed upon by both parties to the account. B) stock certificates may be delivered in the name of either party. C) orders may be entered by either party. D) in the event of death, the other party assumes full ownership of the account.

B In a JTWROS account, each party has an equal, undivided interest in the account. Upon the death of one party in a two-party account, the other party assumes full ownership of the account. Orders may be entered by either party, and mail may be directed to either party. However, disbursements of cash or securities must be in the name of all parties to the account.

Which of the following debt securities would be most likely to offer a conversion feature into common stock? A) Preferred stock B) A debenture C) A mortgage bond D) Commercial paper

B Invariably, when it comes to convertible debt securities, they are debentures rather than secured bonds. The conversion concept makes no sense with money market securities—they mature in one year or less. Preferred stock is often convertible, but it is an equity security, not a debt security.

If a customer buys callable municipal bonds, Municipal Securities Rulemaking Board (MSRB) rules state that the confirmation sent to the customer must disclose A) the nominal yield only. B) the lower of either the yield to call or yield to maturity. C) the higher of either the yield to call or yield to maturity. D) the yield that would result if the bonds were called midway between the date they become eligible to be called and their maturity date.

B MSRB Rule G-15 states that the confirmation must indicate the lowest possible yield.

Which of the following securities is the least suitable recommendation for a qualified retirement account plan account? A) Blue-chip common stock B) Investment-grade municipal bond C) Treasury bill D) A-rated corporate bond

B Municipal bonds provide tax-exempt interest payments and, consequently, offer lower yields. Because earnings in a qualified retirement plan account grow tax deferred, the municipal bond is not a suitable investment. In addition, they will be fully taxed upon withdrawal.

A structured instrument known as an asset-backed security would not be backed by A) student loans. B) loans on marginable securities. C) auto loans. D) credit card debt.

B One common theme uniting asset-backed securities is the contractual obligation to make payments. In the case of a margin account, there is no repayment schedule. The margin debt can exist for years with the only payment being that of interest.

To comply with the regulations regarding customer identification programs, the minimum identifying information that must be obtained from each customer before opening an account includes which of the following? Their name Oral assurance that the customer is of legal age A street address, unless the primary mailing address is a post office box located in the state of residence A taxpayer identification number A) I and IV B) III and IV C) II and III D) I and II

B Oral assurance that the customer is of legal age is not sufficient; the actual date of birth must be obtained. A post office box is never acceptable without a physical address. In addition, the identity of the person opening the account must be verified through documentation such as an unexpired drivers license or passport.

When market interest rates are rising, the market price of which of the following securities would probably be affected the most? A) DEF Technologies, a cybersecurity firm B) ABC Gas and Electric, a regulated public utility C) JKL Corporation adjustable-rate preferred stock D) GHI Money Market mutual fund

B Stocks that are interest rate sensitive will reflect the impact of a change to market interest rates more than others. Utility stocks, with their high degree of debt leverage and liberal dividend payout ratios, are considered interest rate sensitive

One of the critical terms used in the discussion of a wash sale is substantially identical. Not wishing to run afoul of the wash sale rule, an investor selling shares of XYZ common stock at a loss would need to avoid purchasing which of the following securities within the 30-day restricted period? A) XYZ callable preferred stock B) XYX convertible debentures C) XYZ convertible preferred stock D) XYZ cumulative preferred stock

B Substantially identical securities include those convertible or exchangeable into the common stock sold at a loss. That would be case with the XYZ convertible preferred stock. XYZ convertible debentures would also be in that category, but did you notice these were XYX not XYZ?

A Treasury bond is quoted in The Wall Street Journal as follows: Bid 100:15 Asked 100:17 Bid Chg. -1 Yield 7.9 From this information, you know that the nominal yield is A) less than 7.90%. B) greater than 7.90%. C) 7.89%. D) 7.90%.

B The Bid and Asked prices show that the Treasury bond is being quoted at a premium (above par), with a yield to maturity of 7.9%. When bonds are trading at a premium, the nominal yield (coupon rate) is greater than the yield to maturity.

All of the following statements regarding negotiable jumbo certificates of deposit are true except A) they are usually issued in denominations of $100,000 to $1,000,000. B) they are fully insured in any denomination by the FDIC. C) they usually have maturities of less than one year. D) they are readily marketable.

B The FDIC insures only up to $250,000.

A registered representative has just been disciplined under the Code of Procedure. Within how many days does the U-4 need to be updated? A) 60 B) 30 C) 20 D) 10

B The U-4 must be updated within 30 day of a qualifying event, such as a change to the residence address or a disciplinary action.

Which of the following statements regarding both traditional and Roth IRAs is true? A) Contributions are tax deductible. B) Contribution limits are the same. C) Withdrawals at retirement are tax free. D) Distributions must begin in the year after the owner reaches age 72.

B The common factor for both traditional and Roth IRAs is that contribution limits are identical. A significant difference between the two is that Roth IRAs do not have RMDs.

A customer purchases an ABC $100 par 6½% convertible preferred stock at $80. The conversion price is $20. If the common stock is trading 2 points below parity, the price of ABC common is A) $18. B) $14. C) $12. D) $16.

B The conversion ratio is computed by dividing par value by the conversion price ($100 par ÷ $20 = 5). Parity price of the common stock is computed by dividing the market price of the convertible by the conversion ratio ($80 ÷ 5 = $16). $16 − 2 = $14.

The predictions of the VIX Index reflect expected change over A) the previous 30 days. B) the next 30 days. C) the next week. D) the next 91 days.

B The current VIX Index value quotes the expected annualized change in the S&P 500 Index over the upcoming 30 days, as computed from options-based theory and current options-market data.

An investor has unexpectedly received $30,000 from an old debt he had written off. This money will come in handy for a business venture planned for three years from now. Meanwhile, he would like to generate some income on the money with as little risk and as little expense as possible. Which of the following recommendations is likely to be the most suitable for this customer? A) Class A shares of the MNO High-Yield Bond Fund B) Class C shares of the ABC Investment-Grade Bond Fund C) Class B shares of the ABC Investment-Grade Bond Fund D) Class B shares of the XYZ Growth Fund

B The customer wants income with as little risk as possible, so our answer must be one of the choices that offer an investment-grade bond fund. Of those offered, Class C shares would be best, because the customer would pay no front-end sales charge and no CDSC after a short time, probably one year. He will pay somewhat higher 12b-1 fees than with Class A shares, but this will amount to only a fraction of 1% per year, and only for the three years of his investment.

Which of the following statements regarding corporate zero coupon bonds are true? Interest is paid semiannually. The discount is in lieu of periodic interest payments. The discount must be accreted and is taxed annually. The discount must be accreted annually with taxation deferred until maturity. A) II and IV B) II and III C) I and IV D) I and III

B The investor in a corporate zero coupon bond receives the return in the form of growth of the principal amount over the bond's life. The bond is purchased at a deep discount and redeemed at par at maturity. That discount from par represents the interest that will be earned at maturity date. However, the discount is accreted annually, and the investor pays taxes yearly on the imputed interest.

When opening a margin account for an individual customer, there are a number of documents required. Which one of the following never requires the customer's signature? A) The hypothecation agreement B) The margin risk disclosure document C) The loan consent agreement D) The credit agreement

B The margin risk disclosure document must be furnished to the margin account customer, but there is nothing for the customer to sign. A clue to that being the correct choice is that it is the only one without the word agreement. It is agreements that must be signed. Although the loan consent agreement is optional, if the customer wishes to permit the lending of their securities, the agreement must be signed.

A client with an options account contacts the registered representative handling the account with instructions to open the following spread: Buy 1 ABC 100 call and Sell 1 ABC 105 call at a 5-point debit. Under FINRA rules, this order A) is for a bull call spread. B) should be refused. C) will be executed at the next available trade meeting the 5-point limit. D) should be turned in immediately.

B The order should be refused because it is impossible for it to be profitable. This is a bull call spread (but that is not the correct answer here because it has nothing to do with FINRA rules) and will become profitable when the spread widens. With strike prices of 100 and 105, it can never widen more than 5 points.

A technology fund manager concerned about a downturn in the value of his portfolio would hedge by A) selling narrow-based index calls. B) buying narrow-based index puts. C) selling broad-based index calls. D) buying broad-based index puts

B The portfolio consists of sector-specific securities, so broad-based index puts such as the OEX would not be appropriate. Instead, the manager should buy narrow-based index puts (for example, indices on technology and electronics).

Which of the following activities are disallowed under FINRA rules? Opening an account for a 16-year-old individual Accepting a sale in a joint account from one of the owners and having the check payable in the name of that individual Accepting a sale order from the husband only in a joint account owned by both husband and wife Requiring written discretionary authorization before accepting orders for a discretionary account A) II and III B) I and II C) I and IV D) III and IV

B The question is asking for the nonallowable practices. We cannot open an account for a minor, nor may we make a check payable to only one of the parties in a joint account.

A municipal dollar bond is quoted at 98¼ to 98¾. The municipal dealer's spread is equal to $5.00 $50.00 5 basis points 50 basis points A) II and III B) I and IV C) I and III D) II and IV

B The spread is half a point. In each point, which is worth $10, there are 100 basis points. Therefore, half a point is worth $5 and represents 50 basis points.

Gargantuan Computers, Inc., (GCI) conducts a rights offering to its current shareholders at $50 per share, plus one right. If the current market price of GCI is $70, what is the value of one right before the stock trades ex-rights? A) 3 B) 10 C) 5 D) 15

B The stock is trading cum rights (before the ex-date). The formula to calculate the value of one right before the ex-date is follows: CMV minus subscription price divided by the number of rights to purchase one share plus 1. Therefore, one right is valued at $10, computed as ($70 − $50) / 2 = $10.

Before effecting a penny stock transaction with a customer, the member firm must A) provide the customer with the price of the most recent trade in the stock. B) provide the customer with a current bid and asked quote on the stock. C) verify that the customer has sufficient funds in the account. D) receive the signed copy of the risk disclosure document.

B To avoid price gouging, SEC Rule 15g-3 requires that no penny stock transaction may take place without the member firm providing the customer with the current inside market quotes

Before effecting a penny stock transaction with a customer, the member firm must A) receive the signed copy of the risk disclosure document. B) provide the customer with a current bid and asked quote on the stock. C) verify that the customer has sufficient funds in the account. D) provide the customer with the price of the most recent trade in the stock.

B To avoid price gouging, SEC Rule 15g-3 requires that no penny stock transaction may take place without the member firm providing the customer with the current inside market quotes.

A customer invests $20,000 in a direct participation program and signs a recourse note for $50,000. During the first year of operation, the customer receives a cash distribution of $15,000 from the partnership. At year's end, the customer receives a K-1 statement reporting his share of partnership losses of $75,000. How much of the loss may the customer deduct from passive income? A) $35,000 B) $55,000 C) $75,000 D) $0

B To determine basis, add the original investment ($20,000) to any recourse debt assumed by the investor ($50,000). Recourse debt adds to basis as the partner is liable for this amount. Cash distributions received reduce basis ($15,000). At year's end, the investor's basis and the amount he can deduct from passive income is $55,000.

A 7% convertible debenture is selling at 101, and it is convertible into the common stock of the same corporation at $25. The common stock is currently trading at $23. What is the parity price of the debenture? A) $929 B) $920 C) $850 D) $910

B To determine the parity price of the bond, first find the number of shares the debenture is convertible into (conversion ratio) by dividing par value by the conversion price ($1,000 / $25 = 40 shares). Next, multiply the current price of the common by the conversion ratio. The result is the parity price of the bond (40 shares × $23 = $920).

AMZ Corporation has declared a cash dividend of $1.10. On the ex-dividend date, an open order to buy AMZ at 52 stop would A) be automatically adjusted to 53.10 stop. B) remain at 52 stop. C) be automatically adjusted to 52.10 stop. D) be automatically adjusted to 51.90 stop.

B When a stock goes ex-dividend, the price of the stock falls by the amount of the dividend. This would require an adjustment in the amount of the dividend to orders placed below the market. This is a buy stop order and those are placed above the market so there is no adjustment to the price. Remember the SLoBS, where the Sell Limits and Buy Stops are above the market, and BLiSS, the Buy Limits and Sell Stops are below the market

An investor anticipating a rise in interest rates would likely purchase A) variable-rate demand obligations or reset bonds. B) bonds issued by the U.S. Treasury. C) callable bonds. D) corporate bonds.

B When interest rates increase, the market price of all fixed-income securities declines. In the case of variable-rate or reset bonds, the interest rate on those is adjusted based on the movements of market interest rates. As a result, when interest rates increase, the rate paid by the variable rate security increases as well.

ABC Corporation has outstanding a 7.75% convertible debenture currently trading at 102. The bond is convertible into common stock at $40. ABC stock is trading $45 per share. Which of the following statements is true? A) To profit in this situation, the investor should buy the stock and short the bonds. B) To profit in this situation, the investor should buy the bonds and short the stock. C) The bond is at parity with the stock. D) An arbitrage opportunity does not exist in this situation.

B With a conversion price of $40, the bond is convertible into 25 shares of ABC common stock ($1,000 / $40 = 25 shares). As the common stock is currently trading at $45 per share, the value of the stock as converted would be $1,125 (25 shares × $45 = $1,125), which is greater than the current price of the bond ($1,020). Therefore, the bond and the stock are not at parity. An investor could profit in this situation by shorting the stock and buying an equivalent number of bonds. A bond could be purchased for $1,020 and immediately converted into stock worth $1,125—a risk-free profit opportunity.

A margin account with a short credit balance of $39,000 will receive a call for additional funds if the SMV rises above A) $40,000. B) $30,000. C) $20,000. D) $10,000.

B With the maintenance call, the investor has to return the account to the margin maintenance requirement (30% of CMV here). Dividing the credit balance by 1.30 will give the call level: $39,000 divided by 1.30 equals $30,000.

You are at a social gathering speaking with an individual who is a tenured professor of astrophysics at the state university. She mentions that she participates in the school's TSA plan. That means she A) is a participant in the teacher-student-administration plan for school betterment. B) is participating in a retirement plan likely offering tax sheltered annuities. C) has qualified for additional compensation because she has earned tenure. D) is training employees of the Transportation Security Administration.

B stands for tax-sheltered annuity and is the most common name for the 403(b) retirement pla

If a customer buys 200 XYZ at $58 and writes 5 XYZ Jan 60 calls at 2, the maximum potential loss is A) $12,600. B) unlimited. C) $1,400. D) $10,600.

B This is an example of ratio writing. Short calls are covered by long stock, but in this example, it would take 500 shares of stock to cover the five calls written. Because there are three uncovered calls, the maximum loss is unlimited.

DMF Company has $50 million of convertible bonds (convertible at $50) outstanding. The current market value of DMF's stock is $42. The bond indenture contains a nondilution feature. If DMF declares a 10% stock dividend, the new conversion price will be A) the stock's current market price. B) lower than $50. C) higher than $50. D) $50.

B With an antidilution feature, the issuer will increase the number of shares available upon conversion if the company declares a stock split or stock dividend. This means the bondholder must be able to convert it to more shares, which requires a lower conversion price.

To be designated as an accredited investor under Regulation D, a married couple must have an income in excess of A) $500,000 for the past two years with an expectation of reaching that level again this year. B) $300,000 for the past two years with an expectation of reaching that level again this year. C) $100,000 for the past two years with an expectation of reaching that level again this year. D) $200,000 for the past two years with an expectation of reaching that level again this year.

B for couples, D for individuals

Regulation T permits borrowing money for the purchase of each of the following except A) listed stocks and bonds. B) listed options with expirations of less than nine months. C) unlisted stocks and bonds. D) listed warrants.

B-Options with expirations of less than nine months must be fully paid without exception. With some exceptions, warrants, stocks, and bonds may be purchased on margin.

Three friends plan to start a new business. It is anticipated it will be several years before the business turns a profit. Which of the following types of business organization would be best if they wish to limit their liability while, at the same time, being able to receive favorable tax treatment for the expected losses? A. C Corporation B. S Corporation C. General partnership D. Sole proprietorship

B. The only way to limit liability is through a corporation(or LLC or limited partnership). The S Corp allows for the flow through of operating losses to shareholders. While the C corporation does not.

An institutional customer would like to use one broker-dealer to handle the administration of the account but would like to use various other BDs to execute trades for certain types of securities. Which type of account would meet the customer's needs? A. An Advisory account B. A Prime Brokerage Account C. A Fee Based Account. D A DVP account

B. A prime brokerage account is one in which a customer, generally an institution selects one member firm to provide custody and other services while other firms, called executing brokers, handle most of the trades placed by the customer

A bank doing which of the following would not be required to register as a municipal broker-dealer with FINRA? A) Underwriting municipal securities for municipal issuers B) Engaging in transactions to purchase or sell municipal securities for public customers C) Holding municipal securities as custodian for public customers D) Providing investment research regarding municipal securities to public investors

C

A bond investor who is looking for capital gains should invest in bonds when interest rates are A) high and expected to rise. B) low and expected to rise. C) high and expected to decline. D) low and expected to decline.

C

A customer buys a municipal bond in the secondary market at 96 that has four years to maturity. Two years later, the customer sells the bond at 99. The tax consequences of this investment are A) three points of ordinary income. B) two points of capital gain and one point of ordinary income. C) two points of ordinary income and one point of capital gain. D) three points of capital gain.

C

A customer establishes the following positions: Long 1 ABC Jun 25 call at 2 Long 1 ABC Jun 25 put at 2 At expiration, the position is profitable if the stock price is A) above 21 or below 29. B) above 25 or below 25. C) below 21 or above 29. D) greater than 21.

C

A customer of a member firm has just invested $100,000 into an equipment leasing DPP. Under FINRA rules, the maximum compensation allowable to the firm is A) $15,000. B) $2,000. C) $10,000. D) $5,000.

C

A customer tells a broker to buy 1,500 shares of ABC at 33.60 immediately for the full 1,500 shares. This is A) an immediate-or-cancel (IOC) order. B) a good-til-canceled order. C) a fill-or-kill (FOK) order. D) an all-or-none (AON) order.

C

A designated market maker is permitted to do all of the following except A) buy and sell for a proprietary account. B) accept a limit order. C) accept a not-held order. D) represent a bid and offer simultaneously.

C

A direct participation program (DPP), organized as a limited partnership, must avoid at least two characteristics of a corporation. Which two characteristics are the easiest to avoid? A) Centralized management and continuity of life B) Continuity of life and decentralized management C) Continuity of life and freely transferable interests D) Freely transferable interests and centralized management

C

A registered representative is arrested for a felony. This arrest A) requires the firm to terminate the representative's registration. B) has no impact on the representative's employment. C) must be reported on an amended Form U4. D) must be dismissed before the representative may continue to act in a registered capacity.

C

A unit investment trust has 90% of its portfolio invested in high-grade bonds with an average maturity of almost 25 years. If the industry consensus was that long-term interest rates were about to increase sharply, which of the following actions would most likely be taken? A) Ladder the maturities B) Shorten the average maturity by moving into short-term bonds C) No action would be taken D) Liquidate the long-term bonds and begin to move into cash or cash equivalents

C

After a company splits its stock 2 for 1, an investor who owns 100 shares receives A) another certificate for 200 shares. B) notice to send in the current certificate to be replaced by a new certificate for 200 shares. C) another certificate for 100 shares. D) notice that the investor's 100-share certificate now represents 200 shares.

C

All of the following are risks of investing in publicly traded mortgage-backed securities except A) borrowers might default on their mortgage payments. B) rising interest rates might extend the date of repayment of principal. C) the market for mortgage-backed securities is illiquid. D) falling interest rates might accelerate early repayment of principal.

C

All of the following sources of revenue could be used to service general obligation debt except A) sales taxes. B) ad valorem taxes. C) user charges. D) fines.

C

All of the following statements regarding the over-the-counter (OTC) market are true except A) more issues trade OTC than on the exchanges. B) it trades listed securities. C) it is an auction market. D) it trades unlisted securities.

C

All of the following terms are associated with general obligation (GO) bonds except A) coterminous debt. B) limited tax bond. C) protective covenants. D) voter referendum.

C

All open orders must be confirmed to the order book A) April 1 and October 1. B) every six months after the order has been entered. C) the last business days of April and October. D) once a year on the anniversary of the order.

C

All the following retail communications must be prefiled with FINRA except A) retail communications concerning options without previously providing an ODD. B) retail communications concerning investment companies with custom ratings. C) retail communications concerning public DPPs. D) retail communications concerning the member firm's opening for business last month.

C

An important basic characteristic of common stocks that makes them a suitable type of investment for the separate account of variable annuities is A) the yield is always higher than mortgage yields. B) the safety of the principal invested. C) changes in common stock prices tend to be more closely related to changes in the cost of living than changes in bond prices. D) the yield is always higher than bond yields.

C

An investor purchased 100 shares of Wilmont Auto Supply Holdings (WASH) on June 1, 2018, at $55 per share. On July 5, 2019, WASH is trading at $40 per share and the investor sells at the market price. On August 1, 2019, the investor purchases a WASH Jan 40 call @4. If there are no other transactions during 2019, the investor's tax consequences are A) not able to be determined until we know the disposition of the option. B) $1,500 short-term loss because the option was purchased less than 30 days after the sale erasing the holding period. C) no taxable loss for 2019 because of the wash sale rule. D) $1,500 long-term loss with no wash sale violation because the stock was long term before the purchase of the option.

C

An investor who purchases 20-year Aaa rated corporate zero-coupon bonds would be least concerned with A) default risk. B) purchasing power risk. C) reinvestment risk. D) interest rate risk.

C

Arbitration and mediation are two services provided by FINRA to settle disputes between members. Regarding these services, which of the following statements are not true? Mediation is mandatory; arbitration is not. Arbitration always results in a binding decision; mediation may not. If arbitration is unsuccessful, the dispute moves on to mediation. A mediator in a dispute may not serve as an arbitrator in the same dispute. A) II and III B) II and IV C) I and III D) I and IV

C

At expiration, if the market price of the underlying common stock and the strike price are the same, all of the following customer positions will show a profit except A) short calls. B) short puts. C) long straddles. D) short straddles.

C

Cash dividends from real estate investment trusts (REITs) are A) taxed as long-term capital gains. B) taxed at a maximum rate for qualified dividends. C) taxed as ordinary income. D) not taxed.

C

Each of the following can change the special memorandum account (SMA) balance in a long margin account except A) market appreciation of securities. B) sale of securities. C) decrease in value of securities. D) cash dividends deposited.

C

If a municipal bond rated BBB is prerefunded, all of the following statements are true except A) the rating of the issue will increase. B) funds required to meet debt servicing have been set aside in escrow. C) the marketability of the issue will decrease. D) the issue is now backed by U.S. government securities.

C

If a prospectus for a variable life insurance product contains hypothetical projections of returns, A) they may be used to demonstrate why this is an investment product. B) they must reflect returns for the past 1-, 5-, and 10-year periods. C) the maximum return permitted is 12%, and there must be an illustration showing a 0% return as well. D) the issuer could be liable for civil action.

C

If a state agency has issued a moral obligation bond that runs into difficulty and requires the secondary backing to be implemented, what is necessary? A) Voter approval B) Administrative approval C) Legislative approval D) MSRB approval

C

If an investment representative hosts an investment seminar and intends to discuss general investment concepts and a specific mutual fund for which he has performance charts, which of the following are true? He may discuss the investment returns of the mutual fund in general, provided he does not use a specific time frame. He may discuss the investment returns of the mutual fund using a specific time frame. He must disclose all material facts regarding the mutual fund to the audience. He may emphasize the positive aspects of the mutual fund and refer prospective investors to the prospectus for further details. A) I and IV B) I and III C) II and III D) II and IV

C

Index options differ from equity options in which of the following ways? A) The trade settlement date is the next business day. B) They are subject to closing purchases as well as closing sales. C) The exercise settlement is in cash. D) The expiration date is the third Friday of the expiration month.

C

New issues of municipal securities are available in which of the following forms? Bearer Book entry Registered Registered as to principal only A) I and III B) II and IV C) II and III D) I and IV

C

Nonstatistical factors used in comparing one mutual fund to another would include all of the following except A) reinvestment privileges. B) withdrawal plan options. C) management fees. D) conversion or exchange privileges.

C

On Friday, September 15, an investor goes long 1 OEX Dec 575 call at 7 when the index is at 581.96. At expiration, the investor closes out the long position at intrinsic value when the index is at 580. What amount of money will be deposited in the investor's account on the following Monday, and what will the profit or loss be to the investor? A) $500 is deposited and there is a $200 gain for the investor. B) $696 is deposited and there is a $196 gain for the investor. C) $ $500is deposited and there is a $200 loss to the investor. D) $500 is deposited and there is a $196 loss to the investor.

C

One of your customers has recently celebrated a 58th birthday. The investor began a regular investment program into shares of the KAPCO Growth Fund over twenty years ago. The account is showing a substantial gain. Because retirement is getting closer, you suggest using the exchange privilege offered by the KAPCO fund group. Your recommendation is to place half of the holdings into the KAPCO Balanced Fund. Following this recommendation would result in A) tax deferral of any gains because the investor has not received any proceeds. B) a taxable transaction for those shares exchanged plus a 10% tax penalty for early withdrawal. C) a taxable transaction for those shares exchanged. D) tax deferral of any gains because the money is still in the KAPCO fund group.

C

Prior to effecting an initial penny stock transaction for a new customer, the registered representative must do all of the following except A) obtain a signed risk disclosure document from the customer. B) obtain a signed suitability statement from the customer. C) determine if the client has been receiving monthly statements. D) determine suitability based on financial condition, investment experience, and investment objectives.

C

Programs allowing for the direct pass-through of losses and income to investors include all of the following except A) S corporations. B) oil and gas drilling direct participation programs. C) real estate investment trusts (REITs). D) new-construction real estate direct participation programs.

C

Recent years have shown an enormous growth in the sales of exchange-traded funds (ETFs). Some of the benefits of using ETFs in your clients' portfolio would include A) lower risk than most comparable mutual funds. B) greater management flexibility than most comparable mutual funds. C) greater tax efficiency than most comparable mutual funds. D) greater diversification than most comparable mutual funds.

C

Regulation SHO prohibits A) short sales on an uptick. B) short selling in cash accounts. C) naked short selling. D) unsuitable short selling.

C

Shares of Marc Lighting Manufacturing Company trade on the NYSE with the ticker symbol MLM. The company wants to raise equity capital by issuing additional shares. An investment bank buys MLM's new shares and sells them to interested retail and institutional buyers. This transaction is most accurately described as A) a best-efforts offering. B) underwriting an IPO. C) an additional primary offering. D) a private placement.

C

TCB Corporation wants to offer $75 million worth of common stock solely to residents of its home state. The issue will not be registered at the federal level. What type of registration will TCB use to register with the state? A) Coordination B) Notice filing C) Qualification D) Regulation D

C

The ABC Corporation would like to raise capital via a Regulation D private placement. Under Rule 506(c), which of the following statements is true? A) Under Rule 506(c), a prospectus is only required for nonaccredited investors. B) Private placements under Regulation D cannot be publicly advertised. C) If the offering is limited to accredited investors, advertising is permitted. D) If the offering is limited to no more than 35 nonaccredited investors, advertising is permitted.

C

The bond placement ratio, as shown in The Bond Buyer, is computed by taking A) the dollar amount of new issues sold divided by the dollar amount of new issues unsold. B) the number of new issues unsold divided by the number of new issues offered. C) the dollar value of new issues sold divided by the dollar amount of the new issues offered. D) the number of new issues divided by the 30-day visible supply.

C

The call provisions of a municipal issue would be detailed most completely in A) the legal opinion. B) the official notice of sale. C) the bond resolution. D) The Bond Buyer.

C

The manager will credit each syndicate member based on sales of that particular maturity allotted to the member, and such credits shall extinguish liability based only on such securities that are sold by the member. This statement describes an agreement among underwriters that is A) an Eastern account. B) an undivided account. C) a divided account. D) a proportionate underwriting.

C

The reoffering yield on a new municipal bond issue is A) the tax-equivalent yield of the new issue. B) the coupon rate on the new issue. C) the yield at which the bonds are offered to the public. D) the interest rate minus any premiums that underwriters are willing to pay.

C

The terms of municipal general obligation (GO) and revenue bond offerings may be set by the issuer as A) neither a competitive bid nor negotiated underwritings. B) negotiated underwriting arrangements only. C) either competitive bid or negotiated underwritings. D) competitive bid underwriting arrangements only.

C

Under FINRA Rule 2273, a registered representative leaving one member firm and licensing with another must follow certain rules when recruiting former clients. Among those is the delivery of a disclosure document in educational format explaining certain issues the current customer should evaluate. Among those issues are all of the following except A) potential costs related to transferring assets to the recruiting firm, including differences in the pricing structure and fees imposed by the customer's current firm and the recruiting firm. B) that some assets may not be directly transferrable to the recruiting firm and, as a result, the customer may incur costs to liquidate and move those assets or account maintenance fees to leave them with his current firm. C) the change will require the customer complete new account forms. D) differences in products and services between the customer's current firm and the recruiting firm.

C

Under the Code of Arbitration Procedure, arbitrators fall into one of two categories: public or nonpublic arbitrators. Which of the following persons could not be a public arbitrator? A) A retail investor who has filed a complaint against a broker-dealer in the last two years B) An attorney who has represented an investor in an arbitration case once in the past five years C) Any person who worked in the financial industry for any portion of her career D) Private investors who have substantial mutual fund holdings

C

Under the Uniform Practice Code, regular way transactions for common stock settle on A) the fourth business day following the trade date. B) the same day as the trade date. C) the second business day following the trade date. D) the next business day following the trade date.

C

Under which of the following terms does the underwriter act in a dealer capacity? A) Selling group B) Syndicate C) Firm commitment D) Best efforts

C

Voter approval may be required for new bond issues for construction of which of the following? Airports Turnpikes State prisons Public high schools A) I and III B) II and IV C) III and IV D) I and II

C

When a FINRA member firm has been designated as a "taping" firm, it is required to A) institute a program of taping all conversations between registered representatives and their supervisors. B) retain copies of all mandated recordings for a period of six years from the date the tape was created. C) retain copies of all mandated recordings for a period of three years from the date the tape was created. D) provide last sale and quote information to the ticker tape promptly after the close of the day's trading.

C

When an outstanding bond issue is the subject of a refunding, the holders of those bonds have their claim on any pledged assets terminated. This is known as A) termination. B) default. C) defeasance. D) replacement.

C

When is the sales charge deducted from purchases of mutual fund shares made under a letter of intent? A) When each letter of intent is completed B) Annually C) When each purchase is made D) Monthly

C

Which of the following actions would cause a corporation's earnings per share (EPS) to increase? A) The exercise of outstanding warrants B) An increase in cost of goods sold (COGS) C) A reduction in the number of shares outstanding D) A 3:2 stock split

C

Which of the following does not participate in the syndicate (joint account) for a municipal underwriting? A) A bank dealing in municipal securities B) A municipal broker-dealer C) The issuing municipality D) A financial advisor acting as a municipal securities dealer

C

Which of the following is contained in an official notice of sale? A) Reoffering yields on the bond B) Agreement among underwriters C) Amount of good-faith deposit required with the bid D) Delivery date

C

Which of the following is defined as profits after taxes and interest paid, less preferred dividends, divided by the number of shares of outstanding common stock? A) Book value per share B) Cash flow per share C) Earnings per share (EPS) D) Price to earnings

C

Which of the following methods of real estate investing is a flow-through vehicle? A) Common stock in a homebuilding company B) A real estate investment trust (REIT) C) A real estate limited partnership (RELP) D) Purchasing a condominium as a personal residence

C

Which of the following municipal issues would least likely involve overlapping debt? A) A library district B) A school district C) An airport district D) A park district

C

Which of the following municipal securities could have been sold in a negotiated underwriting? A) School bonds B) Industrial development bonds C) All of these D) Limited tax bonds

C

Which of the following ratios is normally considered adequate coverage of interest and principal charges for a municipal revenue bond? A) 1:1 B) 3:1 C) 2:1 D) 7.5:1

C

Which of the following risk factors would be least important to disclose in recommending collateralized mortgage obligation (CMO) securities to public customers? A) Prepayment risk B) Extended payment risk C) Credit risk D) Interest rate risk

C

Which of the following statements best describes a breakpoint sale? A) Sale of investment company shares in anticipation of a distribution scheduled to be paid shortly B) Sale of investment company shares in dollar amounts above the point at which the sales charge is reduced C) Sale of investment company shares in dollar amounts slightly below the point at which the sales charge is reduced on quantity transactions, to make a higher commission D) Compensation generated by commissions from a client who has reached another breakpoint, paid to the registered representative after he no longer works for the member

C

Which of the following statements regarding Sallie Mae debentures are true? A) Sallie Mae securities finance building public schools across the country. B) They are backed by the taxing power of the U.S. government. C) Interest is tax exempt at the state and local levels. D) Interest is paid monthly.

C

Which of the following statements regarding discretionary accounts is not true? A) Each discretionary order must be reviewed promptly by a principal. B) The customer must grant written authorization to the broker-dealer or a designated individual to exercise discretion in the account. C) The customer must approve each order before or after it is executed. D) The account may not be accepted unless approved, in writing, by a principal of the member firm.

C

Which of the following statements regarding hedge funds is correct? A) Hedge fund managers, like mutual fund managers, are compensated largely based on assets under management. B) Hedge funds are passively managed in an attempt to provide predictable returns for investors. C) Hedge funds are usually structured as a partnership. D) Hedge funds are typically favored by inexperienced investors to hedge against losses they may experience as they gain investment savvy.

C

Which of the following statements regarding joint accounts registered as tenants in common (TIC) are true? Each party specifies a percentage of interest in the account. Each party has an equal interest in the account. The interest of a deceased tenant passes to the estate of the decedent. The interest of a deceased tenant passes to the cotenant. A) I and IV B) II and IV C) I and III D) II and III

C

Which of the following statements regarding savings incentive match plans for employees (SIMPLEs) is not true? A) SIMPLEs are retirement plans for small businesses with100 or fewer employees. B) Catch-up contributions for those age 50 and older are permitted. C) Employers cannot make matching contributions for employees. D) Employee contributions are pretax.

C

Which of the following underwriting arrangements is associated with an invitation, typically found in The Bond Buyer, directed at investment bankers and broker-dealers, intended to solicit interest in underwriting a new municipal issue? A) Best efforts B) Negotiated C) Competitive bid D) All or none

C

Which of the following would be defined as a research report? A) A notice that the rating for a specific bond has been downgraded by Moody's B) A technical analysis that indicates the demand for steel is increasing based on the trading volume and price of the common stock of XYZ Steel Corporation C) A document that states the banking industry is ready for recovery but ABC Bank will not participate in the recovery and if owned, investors should sell the security D) A written opinion that the economy is poised for recovery

C

Which type of fund is most often organized as a limited partnership? A. Face-amount certificate (FAC) company B. Exchange-traded fund (ETF) C. Hedge fund D. Unit investment trust (UIT)

C

While acting in a financial advisory capacity to a municipal issuer, a municipal securities dealer wants to be part of a syndicate in the underwriting of one of the issuer's new bonds. Which of the following statements regarding this situation is true? A) The dealer must obtain the MSRB's written approval before signing the syndicate letter. B) The dealer would be allowed to participate and collect fees for both advisory and underwriting services supplied. C) This is recognized by the MSRB as a potential conflict of interest; municipal rules generally prohibit a broker-dealer from acting in both capacities. D) Only an approval by the SEC could allow the broker-dealer to function in both capacities.

C

You have just received a statement of claim from the director of arbitration. One of your customers is claiming that your failure to follow his instructions led to a loss of $36,000. Under FINRA rules, you must respond within A) 24 business hours. B) 60 days. C) 45 days. D) the business day the claim was received.

C

hich of the following positions subject an investor to unlimited risk? Short naked call Short naked put Long put Short sale of stock A) I and III B) I and II C) I and IV D) II and III

C

In the trading of options, there are a number of different multiple option strategies. An investor has the following position: Buy one RIF Apr 120 call Buy one RIF Jul 130 put Which strategy is the investor using? A) Diagonal spread B) Time spread C) Long combination D) Long straddle

C A combination is composed of a long call and long put, or a short call and a short put, each having different strike prices and/or expiration months on the same underlying security. A straddle is when the expiration dates and exercise prices are the same. A spread consists of a long and short position in the same options class (two puts or two calls)

Variable-rate municipal bonds are subject to all of the following risks except A) market. B) liquidity. C) interest rate. D) default.

C A variable-rate bond is one whose coupon is adjusted periodically (semiannually or annually) to reflect current interest rates. Therefore, if rates rise and force prices down, the coupon on a variable-rate bond will be adjusted upward, thereby tending to keep the bond's price at or near par. Therefore, no interest rate risk is associated with these bonds. However, if rates fall, the coupon will be adjusted downward, keeping the bond's price at or around par. Normally, a fall in rates will force prices up, but not with variable-rate bonds.

When a customer, who is at least 59½, withdraws money from a traditional IRA that has been funded totally with deductible contributions, A) the withdrawal causes the entire IRA balance to be subject to taxation at ordinary income tax rates. B) the basis is taxed as ordinary income, and the gains are taxed at the capital gains rate. C) the entire amount withdrawn is subject to taxation at ordinary income tax rates. D) the entire amount withdrawn is subject to taxation at ordinary income tax rates with an additional 10% penalty.

C All withdrawals from a traditional IRA that has been funded with pretax contributions are subject to taxation at ordinary income tax rates. There is no 10% penalty once the account holder has reached age 59½.

Growth of capital is most likely to be the investment objective of which of the following mutual funds? A) A money market fund B) A GNMA fund C) An asset allocation fund D) A U.S. government securities fund

C An asset allocation fund allocates the assets in its portfolio among equity, fixed income, and cash. The allocations change in tune with market opportunities. The equities in the portfolio are designed to generate capital growth, and proper timing of fixed income securities can also lead to growth of capital. Although the cash won't grow, moving assets into cash at the right time can reduce the amount of decline in the portfolio. The other choices all focus on income and perhaps stability.

A FINRA member firm making a bulk transfer of customers' assets would most likely give notification through A) FINRA's Central Registration Depository (CRD). B) a broadly circulated publication such as the Wall Street Journal. C) a negative response letter. D) a positive response letter.

C An example of a bulk transfer is the member firm deciding to switch money market funds used for sweeps of customer credit balances. A negative response letter is one where the customer's agreement is assumed unless responding negatively to the change.

A bond investor who is looking for capital gains should invest in bonds when interest rates are A) high and expected to rise. B) low and expected to decline. C) high and expected to decline. D) low and expected to rise.

C As interest rates rise, bond prices fall. Conversely, when interest rates decline, bond prices increase

On March 1, an individual, age 40, wants to open and fund a Roth IRA at the maximum permitted level. She earns less than the adjusted gross income level that would limit her contribution. What is the maximum amount that she may place in a new Roth IRA? A) $7,000 B) $14,000 C) $12,000 D) $6,000

C Based on her age (less than 50), her maximum contribution would be $12,000, specified as $6,000 for two separate years of contributions. Because she is opening the account on March 1, she would be permitted to make contributions for the prior tax year (up until the April 15 tax filing deadline), as well as for the current tax year.

Which of the following statements regarding revenue bonds issued by a state or municipality is true? A) The bonds carry an unqualified promise to pay interest and principal backed by the power of the issuer to levy taxes. B) Interest and principal payment is backed by the full faith and credit of the issuer. C) Interest will be paid only if the enterprise owned and operated by the state or municipality has sufficient earnings to cover the interest payments or the debt service reserve. D) Interest and principal payment is guaranteed.

C Because revenue bonds are not backed by the full faith and credit of the municipality that issues them, the earnings of the revenue-producing project must be large enough to cover the interest and principal payments.

Which of the following are money market securities? Commercial paper Treasury bonds American depositary receipts Negotiable certificates of deposit A) II and IV B) II and III C) I and IV D) I and III

C Commercial paper and negotiable certificates of deposit are short-term debt securities and are considered money market securities.

A successful chain of retail stores in the maximum corporate tax bracket may exclude from taxation 50% of income earned on investments in A) municipal bonds from the same state in which the corporation is located. B) industrial development bonds. C) domestic corporate common and preferred stock. D) government and agency securities.

C Corporate ownership of another company's stock allows the investor to exclude 50% of the dividends from taxation.

Which of the following statements regarding convertible bonds is not true? A) If there is no advantage to converting the bonds into common stock, they would sell at a price based on their market value without the convertible feature. B) Convertible bondholders are creditors of the corporation. C) Coupon rates are usually higher than nonconvertible bond rates of the same issuer. D) Coupon rates are usually lower than nonconvertible bond rates of the same issuer.

C Coupon rates are not higher; they are lower because of the value of the conversion feature. The bondholders are creditors. If the stock price falls, the conversion feature will not influence the bond's price.

Regarding the taxation of dividends received from corporate securities, which of the following are true? Nonqualified dividends are taxed at the rate the investor's ordinary income will be taxed. Nonqualified dividends are not taxed. Qualified dividends are taxed at a maximum rate specified by the IRS and will depend on the investor's income tax bracket. Qualified dividends are taxed at the rate the investor's ordinary income will be taxed. A) I and IV B) II and III C) I and III D) II and IV

C For income tax purposes, corporate dividends are divided into two categories: qualified and nonqualified. We don't expect you'll be tested on what makes a dividend qualified or not, but you will need to know the difference in taxation. Qualified dividends are taxed at the same rate as long-term capital gains—a rate significantly lower than the ordinary income tax rate levied against nonqualified dividends.

A sophisticated client has expressed an interest in becoming more aggressive with their investment strategy. Her current portfolio consists of the following: $50,000 cash $200,000 in retirement accounts $100,000 in various individual stocks in different industries $100,000 in a balance fund She is willing to invest $25,000 for a minimum of 7 to 10 years and accepts that the investment can and will fluctuate in value over time. Which of the following investments would be the most appropriate? A) MNO High-Yield Bond Fund B) XYZ Value Equity Fund C) ABC Capital Appreciation Small-Cap Fund D) DEF Asset Allocation Fund

C For someone who is willing to take the risk and invest for the long haul, a small- or mid-cap growth fund would be appropriate.

A customer purchases an XYZ municipal bond at 108. It is scheduled to mature in 16 years. After owning the bond for 10 years, she sells the bond at 102. What capital gain or loss must she report for tax purposes at the time of the sale? A) $10 gain B) $60 loss C) $10 loss D) $20 gain

C If a municipal bond is purchased at a premium, the premium must be amortized over the time until maturity. An $80 premium on a 16-year municipal bond indicates that $5 will be amortized each year ($80 / 16 = $5). Ten years at $5 per year is $50 of amortization. Therefore, after 10 years, the tax basis would be $1,080 minus $50, or $1,030 (103). Because the sale was for 102 ($1,020), the customer has a $10 loss on one bond.

A customer purchases an XYZ municipal bond at 108. It is scheduled to mature in 16 years. After owning the bond for 10 years, she sells the bond at 102. What capital gain or loss must she report for tax purposes at the time of the sale? A) $20 gain B) $60 loss C) $10 loss D) $10 gain

C If a municipal bond is purchased at a premium, the premium must be amortized over the time until maturity. An $80 premium on a 16-year municipal bond indicates that $5 will be amortized each year ($80 / 16 = $5). Ten years at $5 per year is $50 of amortization. Therefore, after 10 years, the tax basis would be $1,080 minus $50, or $1,030 (103). Because the sale was for 102 ($1,020), the customer has a $10 loss on one bond.

Limited partners assisting the general partner to solicit new investors A) is permitted if done within 90 days of her acceptance as limited partner. B) is permitted if no compensation is paid. C) could jeopardize their limited partner status. D) is permitted if stated in the partnership agreement.

C If limited partners—either individually or as a group—become too involved with the business of the partnership, they could be considered general partners and lose their limited liability.

One of your customers is a married couple with a joint account. Spouse 1 owns 9% of the common shares of XYZ. Spouse 2 owns 2% of the common shares of XYZ and wishes to sell some of the XYZ shares. Which of the following statements is correct? Spouse 2 is considered an affiliate. Soouse 2 is not considered an affiliate. Spouse 2 must file a Form 144 to sell. Spouse 2 does not have to file a Form 144 to sell. A) I and IV B) II and IV C) I and III D) II and III

C If spouses (either individually or jointly) own a combined total of 10% or more of a corporation's voting shares, they are considered affiliates and are subject to the requirements of SEC Rule 144. Among those requirements is the filing of Form 144. For exam purposes, spouses are assumed to live in the same home unless the question states otherwise.

A customer asks your advice regarding a deferred compensation plan at work. You should state that A) if the business fails, the employee is considered a secured bondholder in terms of liquidation priority and getting paid back. B) deferred compensation plans usually benefit younger employees because the money in the plan has more time to grow prior to retirement. C) deferred compensation plans may be somewhat risky because the employee covered by the plan becomes a general creditor if the business fails. D) if they sit on the board of directors and are also an employee of the company, they are not eligible for the plan.

C If the business fails, the employee can lose everything they put in the plan. The participant will become a general creditor of the company and will be on the same level as unsecured bondholders in the liquidation priority.

All of the following subject an investor to unlimited risk except A) 1 ABC uncovered (short) call. B) short 100 ABC, write 1 ABC put. C) short 100 ABC, buy 1 ABC call. D) short 100 shares ABC stock.

C Investors use long calls to protect short stock positions. If the market value of the stock needed to cover the short position begins to rise, the investor can exercise the long call position to buy the stock. Short stock positions, short uncovered calls, and short stock combined with short puts all subject investors to unlimited risk.

Opening a margin account involves a number of different documents. The document describing how the interest on the margin debt is calculated is generally known as A) the risk disclosure document. B) the hypothecation agreement. C) the credit agreement. D) the loan consent agreement.

C It is the credit agreement, sometimes referred to as the margin agreement, that describes the creditor-debtor relationship.

One of the major concerns of the regulatory bodies is the growing problem of senior exploitation. To combat this issue, at the time of an account opening, whether margin or cash, a member firm must disclose in writing that the member or an associated person is A) required to meet the requirements with respect to opening a new account, including obtaining the contact information of a trusted person for any account, retail or institutional. B) unable to further update the customer record with regard to a trusted contact person without written consent from the contact person. C) authorized to contact the trusted contact person provided on the new account form by the customer and to disclose information about the customer's account in addressing possible exploitation. D) unable to open the account in the absence of the name and contact information for a trusted contact person.

C Member firms must obtain the name and contact information for a trusted contact person who is 18 or older. This person may be contacted about the customer's account. This is required for retail but not institutional accounts. At the time of the account opening, the member firm must disclose in writing to the customer that the member or an associated person is authorized to contact the trusted contact person. Contacting is done to disclose information about the customer's account to address possible exploitation. The absence of the name and contact information for a trusted contact person does not prevent a member from opening or maintaining an account for a customer, as long as the member makes a reasonable effort to obtain this information.

Most business development companies (BDCs) are classified as A) a unit investment trust. B) an open-end investment company. C) a closed-end investment company. D) an exchange-traded fund.

C Most BDCs register as closed-end investment companies (CEF) and trade in similar fashion in the secondary markets. Federal law places some restrictions on the investment flexibility of a BDC that are not required of regular CEFs. A major difference between BDCs and the other investment companies is the active role played in the management of the businesses in the portfolio. That is what business development is abouthelping smaller businesses develop into larger ones.

According to FINRA's Uniform Practice Code, which of the following deliveries of stock certificates does not constitute good delivery for a transaction of 540 shares? A) One certificate for 500 shares and eight certificates for 5 shares B) Two certificates for 50 shares, three certificates for 100 shares, and seven certificates for 20 shares C) Nine certificates for 60 shares D) One certificate for 300 shares and 12 certificates for 20 shares

C Nine certificates for 60 shares would not constitute good delivery. The Uniform Practice Code rules regarding good delivery requires delivery of certificates in units of 100, multiples of 100, or in smaller certificate amounts that, when added together, make a stack of 100 shares

An individual purchases a single premium deferred variable annuity. There will be income tax ramifications in all of the following situations except A) during the payout period. B) surrender of the contract. C) death prior to annuitization. D) during the accumulation period.

C One of the features of annuities, fixed and variable, is that there are no taxes during the accumulation phase. However, anytime money comes out of the account, whether when annuitized, surrendered voluntarily, or not (as in death), any earnings are subject to taxation.

Phantom income is a characteristic of A) American depositary receipts. B) convertible bonds. C) zero-coupon bonds. D) preferred stock.

C Phantom income is the term describing income that is not received, but it is taxed. With zero-coupon bonds, the principal payoff at maturity represents receipt of the discount in lieu of periodic interest. However, each year, a portion of that discount is reported to the IRS on Form 1099 OID and is taxed as ordinary income unless the security is a municipal bond.

When discussing a client's finances, which of the following would be of least importance when planning to make a lump-sum investment? A) Year-end bonus B) Expected inheritance C) Current salary D) Winning the lottery

C Salary enables the registered representative to determine the funds available for periodic investment. A lump-sum investment could be made with money from an inheritance, a year-end bonus, or lottery winnings.

The GHI Transportation Company has run into decreased sales and is forced into a bankruptcy liquidation. Which of the following would have the most junior claim? A) Holders of GHI's equipment trust certificates B) Holders of GHI's collateral trust certificates C) Holders of GHI commercial paper D) Holders of GHI's mortgage bonds

C Secured debt (such as the mortgage bond), the collateral trust certificate, and the equipment trust certificate have first priority in the event of a bankruptcy. Commercial paper is a promissory note relying on the creditworthiness of the issuer.

Which of the following transactions must occur in a margin account? Short sale of stock Purchase of stock to cover a short position Long purchase of stock Long sale of stock A) II and IV B) III and IV C) I and II D) I and III

C Short sales must always occur in a margin account because the investor is borrowing stock from the broker-dealer. Covering must take place in the same account.

If the assets of a company did not change, but stockholders' equity declined, it follows that A) retained earnings increased. B) capital surplus decreased. C) liabilities increased. D) liabilities declined.

C Stockholders' equity is assets minus liabilities. If assets stay the same, then an increase in liabilities will cause a decline in equity.

A customer has contributed $1,000 a year for 10 years to his tax-deferred nonqualified variable annuity. The value of the separate account is now $30,000. If the customer takes a withdrawal of $10,000, what are the tax consequences? A) Any tax due is deferred. B) Two-thirds of the withdrawal is taxable as ordinary income. C) The entire $10,000 is taxable as ordinary income. D) There is no tax, as the withdrawal is considered return of capital.

C The $30,000 contract value represents $10,000 of contributions and $20,000 of earnings. When a partial withdrawal is made from an annuity, the earnings are considered to be taken out first for tax purposes (or last-in, first-out). Therefore, ordinary income taxes will apply to the entire $10,000. In addition, if the customer is not at least 59½, there will be an additional tax penalty of 10%.

Which of the following responsibilities did the Municipal Securities Rulemaking Board (MSRB) receive through the Securities Acts Amendments of 1975? Regulation of municipal issuers Establishment of recordkeeping requirements for municipal broker-dealers Enforcement of any municipal regulations it adopts Creation of regulations for participants in the municipal securities secondary market A) II and III B) I and IV C) II and IV D) I and III

C The MSRB creates rules for municipal trading and issues interpretations of its rules. It does not regulate issuers or have any enforcement capability. For broker-dealers, MSRB rules are enforced by FINRA.

If an investor opens a new margin account and sells short 100 shares of ABC at 32.50, with Regulation T at 50%, what is the investor's required deposit? A) $1,625.00 B) $812.50 C) $2,000.00 D) $3,250.00

C The Regulation T requirement is $1,625 ($3,250 × 50%). When selling stock short in a new account, an investor must meet the minimum requirement of $2,000 for any short sales of $4,000 or less. Above $4,000, the deposit is 50% of the short market value.

You are reviewing an investor's balance sheet. Which of the following items would be found on a balance sheet and help you determine the client's net worth? 401(k) balance Credit card balance Monthly income Electric bill A) I and IV B) II and III C) I and II D) III and IV

C The balance sheet reflects a person's net worth by comparing assets and liabilities. A 401(k) balance is an asset and credit card debt is a liability. Income and monthly bills, such as the electric bill, are found on the income statement.

The Municipal Securities Rulemaking Board (MSRB) and FINRA rules requiring uniformity of business practices by member firms may be altered or modified by a mutual agreement between the broker-dealers concerned. There is, however, an exception to modifying the rules when it comes to the rules regarding A) the price and date of delivery. B) the payment of shipping costs. C) the content of confirmations. D) the terms of delivery.

C The contents of confirmations are specified in SEC Rule 10b-10 and followed by the MSRB and FINRA. They cannot be altered by a mutual agreement.

One of your clients owns 300 shares of common stock in a publicly traded corporation. The acquisition cost of those shares was $60,000 and the last trade of the stock was $220 per share. There was a news report that the company was going to pay shareholders a 100% stock dividend. The client wants to know how this dividend will affect the holding. You would respond that the customer will A) now own 150 shares and the market price will be approximately $440 per share. B) now own 600 shares and the market price will be approximately $220 per share. C) now own 600 shares and the market price will be approximately $110 per share. D) still own 300 shares and the market price will be approximately $440 per share.

C The effect of a 100% stock dividend is the same as a 2:1 stock split. The customer will have twice as many shares worth half as much each. That would be 600 shares worth $110 per share for a total value of $66,000. Note that the total value is unchanged from the pre-split value of 300 shares at $220 per share.

You have a client who invested in the PQR Growth Fund 10 years ago and now, as retirement age approaches, asks you about using the exchange privilege to move into the PQR Balanced Fund. The client should know that A) any tax consequences are deferred until the Balanced Fund shares are liquidated. B) the old shares are liquidated at NAV and the new shares are purchased at the POP. C) this exchange is considered a taxable event as of the date of the exchange. D) the exchange qualifies for any breakpoint reduction.

C The exchange privilege allows for an exchange at net asset value (NAV) between funds that are members of the same "family." The exchange is considered a taxable event. Because the exchange is made at NAV, the concept of breakpoint is irrelevant.

Assuming ABC is subject to a 60,000-contract position limit, which of the following customer accounts are in violation of the exchange's position limits? Long 35,000 ABC Jan calls, long 30,000 ABC Jan 08 LEAPS calls Long 35,000 ABC Mar calls, long 30,000 ABC Mar puts Long 35,000 ABC Mar calls, short 30,000 ABC Jan 08 LEAPS calls Long 35,000 ABC Mar calls, short 30,000 ABC Mar puts A) II and IV B) III and IV C) I and IV D) I and II

C The limit is based solely on the number of contracts with the same market sentiment. When aggregating contracts to determine if the position limit has been exceeded, we add together all positions in that security on the same side of the market. That is, we look at contracts that are hoping for a move to the upside and combine them, or we look for contracts that are hoping for a move to the downside and combine them. Long calls and short puts benefit from a move to the upside. The reverse is true of long puts and short calls.

The locate requirement of Regulation SHO for short sales does not apply to A) American depositary receipts traded on the Nasdaq Stock Market. B) over-the-counter equity securities. C) nonconvertible bonds traded on the NYSE. D) preferred stock traded on the NYSE.

C The locate requirement is applicable to all short sales of equity securities. It is unlikely to be tested, but, just in case, for purposes of this rule, convertible bonds are considered equity securities.

Which of the following positions does not expose a customer to unlimited risk? A) Short 200 shares of XYZ B) Short 2 XYZ uncovered calls C) Short 2 XYZ uncovered puts D) Short 200 shares of XYZ and short 2 XYZ puts

C The maximum potential loss on a short put position is the market price declining to zero reduced by the premium. Remember, a stock's price can never go below "worthless." For example, if the investor sold 2 XYZ 90 puts and received a premium of 4 point each, the maximum loss would be $8,600 (worthless stock is put to the writer for $9,000 but the writer received the $400 premium) per contract or $17,200. That is a significant loss, but all of the other positions expose the client to unlimited risk because a loss will occur if the stock price rises and there is no upper limit to a stock's price.

In its notice of sale in The Bond Buyer, an issuer states that it will take into consideration the timing of interest payments when evaluating bids. The issuer will be using which of the following methods in its bid selection? A) Low interest cost B) Real interest cost C) True interest cost D) Net interest cost

C The true interest cost (TIC) method takes into consideration the time value of money. The issuer discounts future interest payments to arrive at a present value.

Which of the following would not be found within the protective covenants for a municipal revenue bond issue? A) Catastrophe clause B) Additional bonds test C) The issue's rating D) Flow of funds

C There are different sources for bond ratings, but they would not be found within the revenue issue's protective covenants. The municipality agrees to abide by the covenants, and a trustee appointed in the bond indenture supervises the issuer's compliance with them. S

An investor purchases a newly issued convertible bond at par. The bond is convertible at $25. Three years later, the underlying common stock is trading at $33 per share. If the investor sells the bond at the parity price, A) the investor has no gain and no loss. B) there is a long-term capital gain of $8 per share. C) there is a long-term capital gain of $320. D) there is a long-term capital loss of $175.

C This question involves several steps. The first is to determine the conversion ratio in shares. A bond convertible at $25 per share has a share conversion rate of 40 shares ($1,000 ÷ $25). The second step is to compute the parity price. That is, what are those 40 shares worth? Multiply 40 shares by $33 per share and that equals $1,320. When the bondholder sells the bonds at parity, that $1,320 is received. The $320 profit over the $1,000 initial cost is a long-term capital gain. An alternative that might be easier for some is to look at the appreciation of the stock. It is $8 per share higher than the conversion price of $25. That represents an increase of 32% (8 ÷ 25). If the bond is at parity with the stock, its price must be 32% higher and that brings us again to the $1,320 price.

A customer purchases ten 8% Treasury notes at 101-16. What is the dollar amount of this purchase? A) $10,116 B) $10,812 C) $10,150 D) $10,015

C Though the denomination of the T-notes purchased is not given, always assume par ($1,000) unless told differently in the question. Remember that government notes and bonds are quoted in 32nds. Therefore, a quote of 101-16 means 101 plus 16/32. 101 plus 1/2 = $1,015; $1,015 × 10 bonds = $10,150.

One of your customers, age 52, wishes to open an IRA. His annual income is more than $200,000 and consists entirely of income from rental real estate and income from a trust fund. What amount may your customer contribute to his IRA this year? A) $5,500 B) $7,000 C) $0 D) $6.000

C To open an IRA, a person needs earned income. Income from rental real estate is passive income, while income from a trust fund is portfolio income. This customer has no earned income.

In a restricted margin account, if a customer fails to pay for a new purchase, the broker-dealer must sell out stock with a value of A) the margin call. B) stock cannot be purchased if the account is restricted. C) twice the margin call. D) three times the margin call.

C Twice the value must be sold out of the account to meet a Regulation T margin call. Cash buys stock in a margin account at a 2:1 ratio; therefore, stock will cover a cash debt at the rate of $2 of stock market value to $1 of cash debt.

FINRA Rule 2231 describes the required frequency of customer account statements. In those cases where there is a highly active customer account, statements must be sent A) semiannually. B) monthly. C) quarterly. D) annually.

C Unless a customer account contains penny stocks, statements are sent quarterly. For statement purposes, the term activity includes the receipt of dividends or interest, but that does not change the quarterly requirement.

A customer creates a long straddle by buying 5 ABC Nov 50 calls and 5 ABC Nov 50 puts, paying a total premium of $3,750. If ABC is at 56.50 at expiration, the customer has A) a loss of $1,000. B) a gain of $1,000. C) a loss of $500. D) a gain of $500.

C When an investor goes long a straddle, the expectation is that the stock's price will be volatile, but the direction (up or down) is uncertain. In this question, the stock's price increased putting the calls in the money. At expiration, the call positions would be closed by selling them for their intrinsic value ($56.50 market value minus the $50 strike = $6.50). With each contract representing 100 shares, the $6.50 equals $$650 and because there are five contracts, the total sale proceeds are $3,250. Compare that to the $3,750 initial cost and the result is a loss of $500.

An arbitration case dealing with a dispute between a customer and a member firm in the amount of $72,000 will be heard by A) one arbitrator, unless FINRA determines three is more appropriate. B) three arbitrators, all of whom must be public. C) one arbitrator, unless both parties agree to three. D) three arbitrators, the majority of whom must be public.

C When the amount in dispute is more than $50,000 and no more than $100,000, the usual number of arbitrators is one. If both parties agree, it can be three.

On September 1, an investor sold 100 shares of KLP Corporation common stock for a loss of $1 per share. On September 15, he purchased a KLP convertible bond with a conversion price of $40. How much of the original loss may he now declare for tax purposes? A) None B) $100 C) $75 D) $40

C With a conversion price of $40, the bond could be converted into 25 shares (1,000 ÷ 40) of KLP common stock. Hence, the investor has bought back the equivalent of 25 shares and may only declare a $75 loss, as the remaining $25 loss will be disallowed.

Your customer has purchased 100 shares of Synovial Lubrication Products (SLP) at $95 per share. The date of the purchase was April 22, 2021. Three months later, the customer purchased one SLP Dec 90 put for 3. At the expiration date of the option, SLP's market price is $101 and the option expires unexercised. The customer sells the 100 shares of SLP stock on June 1, 2022 at $104 per share. This sale results in A) a short-term capital gain of $600. B) a long-term capital gain of $600. C) a short-term capital gain of $900. D) a long-term capital gain of $900.

C he option expired in December 2021. That means the loss is taken that year. Selling the stock at $104 creates a capital gain of $900

Sell order tickets must be A) executed in accordance with the appropriate rules, but not necessarily marked. B) marked only if they are long sales. C) marked as either long or short. D) marked only if they are short sales.

C Every sell order must be marked as either a long sale or a short sale.

One of your customers owns a limited partnership interest in an oil and gas drilling program. The program was successful in finding oil and is expected to operate at a loss for the next year. The loss flowing through to the limited partner is generated by all of these except A) depletion on the sale of oil removed from the ground. B) interest payments on partnership debt. C) principal repayment on partnership debt. D) accelerated depreciation taken on the drilling equipment.

C - Losses occur when expenses exceed revenues. Principal repayments are not an expense. Interest on debt is a deductible expense. Natural resources deplete and the depletion allowance is an expense similar in concept to depreciation, another expense. From a personal standpoint, compare this to your home mortgage−the interest is a deductible expense, but the portion representing payment of principal is not.

For an out-of-the-money equity option expiring in seven months, the time value of the option will most likely be A) less than the option premium. B) greater than the option premium. C) equal to the option premium. D) less than zero.

C - The time value of an option will never be less than zero; it can only be zero or positive.

An over-the-counter (OTC) quote that must be reconfirmed with the OTC trading room before a broker-dealer takes action is A) firm. B) third party. C) representative. D) subject.

D

The bond resolution includes all covenants between A) the issuer and the bond counsel. B) the bond counsel and the bondholders. C) the issuer and the Municipal Securities Rulemaking Board. D) the issuer and the trustee acting for the bondholders.

D

Fee Based Account

Charge a single fee (either fixed or a percentage of assets) instead of commission based charges for brokerage services

A FINRA member firm wishes to encourage its registered representatives to sell more limited partnership DPPs. As an incentive, the firm offers an all-expenses-paid trip to a popular vacation resort for those reaching certain sales targets. FINRA rules provide that A) the target must be based on the total production of associated persons with respect to specific investment company securities distributed by the member. B) sales incentives are limited to gifts that do not exceed $100 in value. C) the member can weight the credits differently for different investment companies. D) the target must be based on the total production of associated persons with respect to all direct participation programs offered by the member.

D

A Western account underwriting of $100 million in municipal bonds is established. A member firm agrees to underwrite 10% of the issue and sells out its entire allotment of $10 million. However, some of the other firms participating in the underwriting are unable to sell their full allocation, and $15 million of the bonds remain unsold. What is the financial obligation of the underwriting firm who sold their entire allotment? A) Pooled responsibility for $15 million B) $150,000 C) $1.5 million D) $0

D

A bank doing which of the following would not be required to register as a municipal broker-dealer with FINRA? A) Providing investment research regarding municipal securities to public investors B) Underwriting municipal securities for municipal issuers C) Engaging in transactions to purchase or sell municipal securities for public customers D) Holding municipal securities as custodian for public customers

D

A bank doing which of the following would not be required to register as a municipal broker-dealer with FINRA? A) Underwriting municipal securities for municipal issuers B) Engaging in transactions to purchase or sell municipal securities for public customers C) Holding municipal securities as custodian for public customers D) Providing investment research regarding municipal securities to public investors

D

A certificate in the name of Smith & Company may be signed A) Smith & Company or Smith & Co. B) Smith & Company. C) Smith & Company, a.k.a. SmithCo. D) Smith & Company, Smith & Co., or Smith and Company.

D

A couple in their early 30s have been married for four years, their disposable income is relatively high, and they are planning to buy a condominium. If they need a safe place to invest their down payment for about six months, which of the following mutual funds is most suitable for these customers? A) ATF Capital Appreciation Fund B) ABC Growth & Income Fund C) XYZ Investment-Grade Bond Fund D) LMN Cash Reserves Money Market Fund

D

A customer establishes the following positions: Buy 100 ABC for 63 Write 1 ABC Jan 70 call for 1 What is the customer's maximum gain? A) 600 B) Unlimited C) 700 D) 800

D

A customer writes two ABC Jul 15 puts at 2 when ABC is 14. If the contracts are closed at a premium of 4 when ABC is 13, the customer has A) a $400 gain. B) a $200 loss. C) a $200 gain. D) a $400 loss.

D

A customer's confirmation for a municipal bond callable at par and quoted higher than the nominal yield would show A) current yield. B) yield to call (YTC). C) coupon yield. D) yield to maturity (YTM).

D

A municipal bond, issued with a covenant that states, "If revenue collections are not sufficient to meet debt service requirements, the issue will be backed by the full faith and credit of the municipality," is known as A) a contingent liability bond. B) a double-barreled bond. C) a Section 8 bond. D) a moral obligation bond.

D

A municipality's net total debt is calculated as A) the total debt plus self-supporting debt plus sinking fund accumulations minus overlapping debt. B) the total debt plus self-supporting debt minus sinking fund accumulations minus overlapping debt. C) the total debt minus self-supporting debt plus sinking fund accumulations plus overlapping debt. D) the total debt minus self-supporting debt minus sinking fund accumulations plus overlapping debt.

D

A retiree is paid an annual amount equal to 30% of the average of his last five years' salary. Which of the following retirement plans offers this type of payment? A) Defined contribution B) Deferred compensation C) Profit-sharing D) Defined benefit

D

All of the following are characteristics associated with equity-linked notes (ELNs) except A) they can be exchange traded or traded over-the-counter (OTC). B) they have final payments at maturity linked to the return of an underlying stock or basket of stocks. C) they are considered to be nonconventional structured investments. D) they are equity securities.

D

All of the following kinds of orders may be turned over to the specialist (designated market maker) for execution except A) stop orders. B) limit orders. C) market orders. D) not-held orders.

D

All of the following statements regarding a municipality's debt limit are true except A) that unlimited general obligation bonds may be issued when a community's taxing power is not restricted by statutory provisions. B) that the purpose of debt limits is to protect taxpayers from excessive taxes. C) that revenue bonds are not affected by statutory limitations. D) that the debt limit is the maximum amount a municipality can borrow in any one year.

D

An investor in a limited partnership generating passive losses can offset these against passive income from other partnerships. rental income from direct investments in real estate. dividends received from listed securities. capital gains from the sale of unlisted securities. A) II and III B) III and IV C) I and III D) I and II

D

An investor opens the following positions: Sell short 100 shares of FAB @72; buy 1 FAB Jun 70 call @5. What is the customer's maximum gain, maximum loss, and breakeven point? A) Maximum gain is $300; maximum loss is $6,700; breakeven point is $67. B) Maximum gain is $6,700; maximum loss is $300; breakeven point is $77. C) Maximum gain is $300; maximum loss is unlimited; breakeven point is $77. D) Maximum gain is $6,700; maximum loss is $300; breakeven point is $67.

D

An investor redeems 200 shares of ABC Fund, which has no redemption fee. If the quote is $12.05 bid $13.01 asked, what amount will the investor receive? A) $1,098.00 B) $2,602.00 C) $2,275.50 D) $2,410.00

D

An order memorandum or ticket must be prepared A) by the close of business on T+1. B) by the close of business on the trade date. C) by the settlement date. D) prior to order execution.

D

FINRA Rule 4530 states that each member shall promptly report to FINRA (but in any event not later than 30 calendar days) after the member knows or should have known of the existence of any of the following, other than when A) the member or an associated person of the member is the subject of any written customer complaint involving allegations of theft or misappropriation of funds or securities or of forgery. B) the member or an associated person of the member is indicted, or convicted of, or pleads guilty to, or pleads no contest to, any felony, or any misdemeanor that involves the purchase or sale of any security in a domestic, military, or foreign court. C) the member or an associated person of the member is barred from becoming associated with any member of any securities, insurance, or commodities self-regulatory organization. D) an associated person of the member is the subject of any disciplinary action taken by the member involving suspension, termination, the withholding of compensation or of any other remuneration in excess of $2,000.

D

FINRA's rule on communications allows member firms to use testimonials in retail communications as long as certain disclosures are made. Among those disclosures is A) indicating that if $100 or more in value is paid for the testimonial, the fact that it is a paid testimonial. B) indicating that if less than $100 in value is paid for the testimonial, the fact that it is a paid testimonial. C) indicating that if any compensation is paid for the testimonial, the fact that it is a paid testimonial. D) indicating that if more than $100 in value is paid for the testimonial, the fact that it is a paid testimonial.

D

For an out-of-the-money equity option expiring in seven months, the time value of the option will most likely be A) less than the option premium. B) less than zero. C) greater than the option premium. D) equal to the option premium.

D

For individual public investors, dark pools of liquidity A) allow them to give an order to their broker-dealer to buy or sell securities while only referencing an account known by a number and not their name. B) prevent them from having their own orders entered on exchanges for execution. C) allow them to enter orders that are sent directly to the trading floors of stock exchanges. D) lessen the transparency of the overall market as volume, quote and price information, and market participant identity is unknown.

D

If a customer of a member firm would like to file a claim under the Code of Arbitration Procedure, the claim will not be eligible once A) two years have elapsed from the occurrence or event giving rise to the claim. B) four years have elapsed from the occurrence or event giving rise to the claim. C) ten years have elapsed from the occurrence or event giving rise to the claim. D) six years have elapsed from the occurrence or event giving rise to the claim.

D

If an investor sells 1 AMF Apr 50 put for 2.50 and buys 1 AMF May 60 put for 7.75, the investor has profit when the spread narrows. the spread widens. both puts are exercised. both puts expire. A) I and IV B) I and III C) II and IV D) II and III

D

If an officer of a bank wants to purchase new issues, which of the following statements is true? A) She may purchase a new issue because anyone is allowed to purchase new issues. B) She may purchase a new issue because no banking rules prohibit it. C) She may not purchase a new issue unless the amount she wishes to purchase is considered small in relation to the total offering. D) She may not purchase a new issue because she is considered a restricted person.

D

If at expiration for XYZ options, XYZ stock closes at 40.15, which of the following open option positions will automatically be exercised by the Options Clearing Corporation (OCC)? A) A customer long 1 XYZ 40 put B) A member firm long 1 XYZ 45 call C) A member firm long 1 XYZ 40 put D) A customer long 1 XYZ 40 call

D

In general, commercial paper, a popular money market instrument, has a maturity not exceeding A) 365 days. B) 90 days. C) 30 days. D) 270 days.

D

JEG Corporation common stock is currently trading at $25 per share. The par value of JEG stock A) is most likely more than $25 per share. B) is most likely less than $25 per share. C) is most likely $25 per share. D) has nothing to do with the current market value of the stock.`

D

KAPCO Manufacturing Corporation declares a 5-for-1 stock split on its outstanding shares of $20 par value common stock. This split will cause A) a change to KAPCO's net worth. B) the dividend per share on KAPCO's preferred stock to be reduced. C) the price of the shares to change to $4 per share. D) the par value of the shares to change to $4 per share.

D

Moody's Investment Grade (MIG) ratings are applied to A) corporate bonds. B) municipal bonds. C) money market instruments. D) municipal notes.

D

One of your customers purchased 1,000 shares of PKZ stock on the day preceding the ex-dividend date. Her account instructions are to have all purchases registered in her name. Several weeks after the payable date, she contacts you with the news that she did not receive the dividend from the issuer. The proper procedure is A) explain to your customer that she purchased the PKZ too late to be entitled to the dividend. B) to have your broker-dealer send a DK notice to the firm representing the seller. C) to commence the reclamation process. D) to have your broker-dealer send a due bill to the firm representing the seller.

D

Paying for a securities transaction with all of the following could raise concern about the possibility of money laundering except A) a postal money order. B) a travelers' check. C) cash. D) a personal check.

D

Responding to the student loan crisis, the SECURE Act now permits qualified withdrawals from Section 529 plans to include payments of A) student loan interest up to a maximum of $10,000 per year. B) student loan principal up to an annual maximum of $10,000 per child. C) student loan interest up to a maximum of $10,000 per family. D) student loan interest up to a lifetime maximum of $10,000 per child.

D

Some analysts use an industrial corporation's balance sheet to arrive at what might be considered the intrinsic value per share of the enterprise. That calculation is known as A) earnings per share. B) net asset value per share. C) retained earnings per share. D) book value per share.

D

The Securities and Exchange Commission regulates all of the following except A) initial public stock offerings. B) the secondary market. C) investment adviser and client relationships. D) intrastate securities offerings.

D

The individual responsible for the overall supervision of all of a firm's options activities on behalf of its customers must be A) an office manager. B) the financial and operations principal. C) the general securities principal. D) a registered options principal (ROP).

D

The market price of a convertible bond depends on all of the following except A) the value of the underlying stock into which the bond can be converted. B) the rating of the bond. C) current interest rates. D) the conversion prices of bonds from similar companies.

D

Under Municipal Securities Rulemaking Board (MSRB) rules, which of the following yields for a callable bond would be shown on a confirmation? A) Yield based on farthest in-whole call B) Yield based on catastrophe call C) Yield based on in-part call D) Yield based on nearest in-whole call

D

Under SEC and FINRA recordkeeping rules, all of the following records must be retained for three years only except A) order tickets. B) a trial balance. C) the subsidiary ledgers. D) the blotter or any record of original entry.

D

Using yield-based options, which of the following hedging strategies offers a bond portfolio manager the greatest protection against rising long-term interest rates? A) Sell 30-year T-bond yield-based calls B) Buy 30-year T-bond yield-based puts C) Sell 30-year T-bond yield-based puts D) Buy 30-year T-bond yield-based calls

D

What action could a corporation take that would result in the forced conversion of an outstanding convertible debt security? A) Reduce the dividends on the common stock to a rate lower than the interest on the debt security B) Exercise the conversion feature when the debt security's conversion value exceeds the call price C) Reduce the coupon rate below the dividend rate on the common stock D) Exercise the call feature when the debt security's conversion value exceeds the call price

D

What type of account allows for the irrevocable transfer of almost any kind of asset, including works of art and real estate, for the benefit of a minor? A) Coverdell ESA B) UGMA C) Tenants in common D) UTMA

D

When a registered representative opens an options account for a new client, in which order must the following actions take place? Obtain approval from the branch manager Obtain essential facts from the customer Obtain a signed options agreement Enter the initial order A) I, II, III, IV B) I, II, IV, III C) II, I, III, IV D) II, I, IV, III

D

Which of the following are considered sources of debt service for general obligation (GO) bonds? Tolls on roads Real estate taxes Revenue generated by a hospital Liquor license fees A) I and IV B) II and III C) I and III D) II and IV

D

Which of the following describes additional paid-in capital? A) Total of all earnings since a corporation was formed, less dividends B) May also be called earned surplus C) Total of all residual claims that stockholders have against the corporation's assets D) The difference between the total dollar amount received from the issuance of common stock and the stock's aggregate par value

D

Which of the following is used to report a bankruptcy filing of a registered representative of a FINRA member firm? A) BF B) ADV C) TIF D) DRP

D

Which of the following securities can be traded in the third market? I. A closed-end investment company II. An exchange-traded fund III. An open-end investment company IV. A unit investment trust A) I and IV B) II and IV C) II and III D) I and II

D

Which of the following statements describing Section 529 plans is true? A) They can only be opened for children under the age of 18. B) Most state college savings plans require either the owner or the beneficiary of the plan to be a state resident. C) The fees associated with them are generally the same from state to state. D) The maximum lifetime contribution varies from state to state.

D

Which of the following statements is an accurate interpretation of FINRA Conduct Rules governing the use of communications with the public? A) All communications with individual clients are considered retail communications. B) All retail communications must be filed with FINRA before first use. C) Sales and product promotion materials distributed to registered representatives and other employees are retail communications and must be submitted for FINRA review, even though such materials are not intended for public distribution. D) Institutional communications need not be preapproved by a principal.

D

Which of the following types of retirement plans would be most beneficial to a young employee of a corporation? A) Keogh plan B) Defined benefit pension plan C) Profit-sharing plan D) Defined contribution pension plan

D

Which of the following would be most likely to require a mandatory sinking or surplus fund? A) A tax anticipation note B) A general obligation C) A public housing authority. D) A water and sewer revenue bond

D

Which of the following would not be found within the protective covenants for a municipal revenue bond issue? A) Catastrophe clause B) Additional bonds test C) Flow of funds D) The issue's rating

D

Your customer is opening a new options account. Which of the following need not occur to open the account? A) The client must agree that any material change in financial status requires the broker-dealer be notified and the options agreement be amended. B) The background and financial information provided by the client must be verified by the client and returned within 15 days of the time the account was approved. C) The registered representative must document that the client has received a current OCC disclosure document. D) The OCC must verify the financial information supplied by the client to ultimately approve the account.

D

Your customer wants to know what portion of earnings one of the companies held in her portfolio has available to pay interest expense on bonds the company currently has outstanding. You would be able to find this information A) on the firm's most recent balance sheet. B) on a firm's income statement by subtracting preferred dividends from EBIT. C) by contacting the IRS. D) on the firm's income statement indicated as earnings before interest and taxes (EBIT).

D

n exercise of the option, the holder of a long call will realize a profit if the price of the underlying stock A) falls below the exercise price minus the premium paid. B) exceeds the exercise price. C) falls below the exercise price. D) exceeds the exercise price plus the premium paid.

D

An investor purchased 100 shares of a stock three years ago at $38 per share. Disappointed with the stock's performance, the investor sells it for $35 per share. Two weeks later, after the company announced higher-than-expected earnings, the investor purchased 100 shares at $44 per share. When this investor decides to sell the newly purchased shares, the cost basis will be A) $41 per share. B) $38 per share. C) $44 per share. D) $47 per share.

D $44 plus $3 equals a new cost basis of $47 per share.

FINRA Rule 2330, which deals with members' responsibilities regarding variable annuities, applies under which of the following circumstances? The initial purchase of a deferred variable annuity. The initial purchase of an immediate variable annuity. The initial subaccount allocations. The initial subaccount reallocations. A) I, II, and III B) I, II, III, and IV C) II and IV D) I and III

D (there is no such thing as initial reallocations). On the other hand, this rule does not apply to reallocations among subaccounts made or to funds paid after the initial purchase or exchange of a deferred variable annuity.

Interest received on a California general obligation bond purchased by a San Francisco resident is exempt from A) capital gains taxes only. B) federal income tax only. C) state income tax only. D) state and federal income taxes.

D A municipal bond is generally exempt from federal and state income taxes in the state in which it was issued. The exemption, or lack thereof, applies to interest, not capital gains.

Because money market instruments are designed to meet the short-term cash needs of issuing institutions, which of the following is not a money market instrument? A) Commercial paper issued by the finance corporation of a major automobile manufacturer B) Municipal construction loan note C) Federal Farm Credit Bank note maturing in one year or less D) Newly issued Treasury notes issued to meet a specific government funding requirement

D A newly issued Treasury note would have a maturity of 2-10 years and would not be considered a money market instrument

Which of the following statements regarding ADRs is not true? A) Key risks to identify include currency and political risks. B) Dividends are received in U.S. dollars. C) ADRs make it easy to own a foreign security. D) Holders generally have voting rights.

D ADRs, with few exceptions, do not have voting rights.

In portfolio theory, the alpha of a security or a portfolio is A) a measure of the variance in returns of a portfolio divided by its average return. B) a measurement of a portfolio's performance versus a standard benchmark such as the S&P 500. C) the risk of the portfolio associated with the factors that affect all risky assets. D) the difference between the expected return of the portfolio, given the portfolio's beta, and the actual return the portfolio achiev

D Alpha is the difference between the expected return of the portfolio, given the portfolio's beta, and the actual return the portfolio achieved. The higher the alpha, the better the portfolio has done in achieving excess or abnormal returns. The risk of the portfolio associated with the factors that affect all risky assets is systematic risk.

Section 28(e) of the Securities Exchange Act provides a safe harbor for certain soft dollar compensation extended from broker-dealers to investment advisers. Which of the following is most likely to be included in that safe harbor? A) Desks remaining after the broker-dealer redesigned its office B) Use of vacant office space in the broker-dealer's facilities C) Meal expenses to attend an investment seminar sponsored by the broker-dealer D) Customized software designed to give clients access to asset allocation programs

D Among the items generally in the safe harbor are those items designed to assist the firm's customers. Customized software that helps clients would be acceptable. Although seminar registration expenses are in the safe harbor, travel and transportation expenses, such as meals and lodging, are not. Rent and office furniture are specifically listed as out of the safe harbor.

Which of the following callable municipal bonds trading on a 7% basis is most likely to be called? A) 7.5% coupon, callable at 105 in 2030 B) 6.5% coupon, callable at 105 in 2030 C) 6.5% coupon, callable at 100 in 2030 D) 7.5% coupon, callable at 100 in 2030

D An issuer will call the higher coupon bonds before calling the lower coupon bonds. Of the two bonds with coupons of 7.5%, the one with the lower call price will likely be called first.

A customer who has $100,000 in government bonds, $35,000 in NYSE-listed equity securities, $4,000 in penny stocks, and no account activity in the last eight months must have a statement sent A) quarterly. B) weekly. C) semiannually. D) monthly.

D Any account that has penny stocks in it, regardless of whether there was activity or not, must be sent a statement at least monthly.

A dispute involving a dollar amount of $50,000 or less is eligible for A. remediation. B. a panel of three arbitrators. C. an appeal. D. simplified arbitration.

D Any dispute involving a dollar amount of $50,000 or less is eligible for simplified arbitration.

An investor purchased 100 shares of ABC common stock at $60 per share on March 2, 2019. At the same time, an ABC Oct 55 put was purchased at $2. On June 2, 2020, the investor sold the stock for $85 per share. As a result, the tax consequences are A) $2,500 long-term capital gain. B) $2,300 short-term capital gain. C) $2,500 short-term capital gain. D) $2,300 long-term capital gain.

D As far as the math, the cost is $62 per share ($60 cost of the stock plus $2 for the put). The sale price was $85, the gain is $23 per share, $2,300.

Your customer, who still works, informs you that she will be funding a variable annuity (VA) you have recommended from two sources: a refinancing of her primary home where she will be able to draw out equity that has built up since it was purchased 15 years ago, and cashing out another VA she recently purchased within the past two years without a lifetime income rider like the one you have recommended. Based only on these facts, the VA recommendation is A) suitable if she has enough equity in the home to fund the variable annuity without cashing out the other VA contract. B) not suitable because a lifetime income rider is only for someone who is already retired. C) suitable regardless of funding sources. D) not suitable.

D Based on the information given in the question, the VA recommendation would not be suitable. Refinancing a home to draw out equity has been identified by FINRA as an abusive sales tactic regarding the sales of VAs. Cashing out life insurance policies or VAs where steep surrender charges are likely to exist, particularly in the earlier years of those contracts, is also considered abusive.

If a customer buys 5 ABC Jan 40 puts and writes 5 ABC Jan 45 puts, which of the following statements are true? The customer profits if the spread widens. The customer profits if the spread narrows. The customer is a bull. The customer is a bear. A) II and IV B) I and III C) I and IV D) II and III

D Because a put is a right to sell, the premium on the 45 puts is higher than that of the 40 puts. The customer is writing the put with the higher premium, so this is a credit spread, and the bullish investor will profit at expiration if the difference between the two premiums narrows as the contracts lose value.

A client, who is a manager of a large pension plan, has recently changed the plan's portfolio weighting from 80% equities and 20% fixed income to 40% equities, 40% short-term Treasury debt, and 20% cash and cash equivalents. More than likely, this is an indication that the client's outlook concerning the market is A) unknown. B) neutral. C) bullish. D) bearish.

D Because the client has reallocated the portfolio into highly conservative assets, one would think the manager is expecting a bear market. This new allocation is an attempt to protect against incurring losses from the anticipated market decline.

A 38-year-old investor places $25,000 into a qualified single premium deferred variable annuity. Twenty years later, with the account valued at $72,000, the investor surrenders the policy. If the investor is in the 25% marginal income tax bracket, the total tax liability is A) $11,750. B) $16,450. C) $18,000. D) $25,200.

D Because this is a qualified annuity, the entire withdrawal is taxable. The surrender value of $72,000 has a cost basis of $0.00. That $72,000 is taxed at the marginal rate of 25%. Furthermore, because the investor is younger than 59½ (38 + 20 = 58), there is the additional 10% penalty tax. Effectively, this is a 35% tax on $72,000.

One concern that FINRA has with fee-based accounts is that they might lead to A) over-trading. B) higher commissions. C) churning. D) reverse churning.

D Churning is the practice of over-trading an account, resulting in higher commissions. Reverse churning occurs when a client in a fee-based account pays more in fees than would be paid in a commission-based account. This is generally the case when the customer trades infrequently.

A deferred compensation plan would be most suitable for A) an employer who wants a retirement plan to benefit the younger employees of the company. B) an employer who wants to provide a plan for all of the firm's employees who are in lower tax brackets. C) an employer who wants employees to make their own set contributions to a plan as an incentive for them to remain with the firm. D) an employer with a few highly paid employees who are near retirement age and want to reduce current taxes.

D Deferred compensation plans are nonqualified plans that allow an employer to select which employees may participate in the plan. These plans are more suitable for highly compensated employees who are just a few years from retirement, allowing them to defer earnings and taxation until then. Offering this type of plan to young employees is less suitable due to the risk that the business could fail or the risk that they may leave the firm before retiring, and thus forfeit any benefit. Defined contribution plans are considered more suitable for those further from retirement.

Recent years have shown an enormous growth in the sales of exchange-traded funds (ETFs). Some of the benefits of using ETFs in your clients' portfolio would include A) lower risk than most comparable mutual funds. B) greater management flexibility than most comparable mutual funds. C) greater diversification than most comparable mutual funds. D) greater tax efficiency than most comparable mutual funds.

D ETFs have fewer taxable events.

Exchange-traded funds (ETFs) have traditionally been based on a specified underlying index. In that respect, they are similar to listed index options. Among the differences between the two products is that ETFs settle in one business day while index options settle in two business days. ETFs settle in two business days while index options settle in one business day. an investor owning an ETF receives the current market price of the ETF when selling the position while an owner of an index put option who exercises the option receives the intrinsic value. an investor owning an ETF receives the intrinsic value of the ETF when selling the position while an owner of an index put option who exercises the option receives the current market value of the index. A) I and IV B) I and III C) II and IV D) II and III

D ETFs settle like any other exchange-traded equity security: T+2. Index options settle T+1. An investor selling a long position in an ETF receives the current market price on the exchange just as with any other listed security. Index options settle with cash being paid in the amount of the intrinsic value of the option.

Income from which of the following is not partially exempt to a corporate investor? A) Common stock B) Preferred stock mutual funds C) Preferred stock D) Convertible bonds

D Fifty percent of dividend income received from investments in common stock and preferred stock is excluded from taxation for a corporate investor. This exclusion applies to dividends from mutual funds where all of the portfolio securities are preferred or common stock.

If a contract calls for a delivery between member broker-dealers of 500 shares of stock, all of the following certificate combinations would be a good delivery conforming with the FINRA Uniform Practice Code except A) 10 certificates for 50 shares each. B) 1 certificate for 200 shares and 1 certificate for 300 shares. C) 5 certificates for 40 shares each and 5 certificates for 60 shares each. D) 10 certificates for 30 shares each and 4 certificates for 50 shares each.

D Good delivery between member firms requires delivery of certificates of 100 shares, in multiples of 100, or the ability to put certificates of less than 100 shares into stacks of 100 shares. Note that the 10 certificates, each for 30 shares, could not be stacked in units of 100

Which of the following is a risk most often encountered when investing in common stock? A) Default risk B) Interest rate risk C) Inflation risk D) Market risk

D Historically, stocks have been a good hedge against inflation while debt instruments have not.

ABC and MNO both have the same market price and shares outstanding for their common stock. If ABC's P/E ratio is higher, that would indicate that A) ABC's sales are higher than MNO's. B) ABC sales are lower than MNO's. C) ABC's net income is higher than MNO's. D) ABC's net income is less than MNO's.

D If ABC's P/E ratio is higher than MNO's, then its earnings (defined as net income ÷ shares outstanding) is lower than MNO's.

A corporation plans to make a public tender for 50% of its outstanding bonds. The price of the tender will be set by A) the trustee. B) the paying agent. C) the transfer agent. D) the issuer.

D In a tender offer, the issuer is offering to buy back all or a portion of the issue at a stated price. The price of the tender is set by the issuer although the issuer may engage an underwriter to help it set the price.

Which of the following would be the most likely unsuitable recommendation for a client whose objective is steady income? A) A bank CD B) A subordinated debenture C) A U.S. Treasury bond D) An income bond

D Income (or adjustment) bonds carry the unique characteristic of requiring payment of interest only when the issuer's income is sufficient. They are used primarily for companies undergoing a financial restructuring, usually after a bankruptcy filing. Don't be fooled by the subordinated debenture. Although it stands last in line of the creditors in the event of a liquidation, that does not mean the investor is not going to get regular interest payments, especially when the debenture is investment grade. Bank CDs typically pay interest quarterly.

Which of the following is not under governance of the Municipal Securities Rulemaking Board (MSRB)? Issuers of municipal fund securities Broker-dealers that sell municipal fund securities Issuers of municipal bonds Banks that sell municipal securities A) I and II B) II and III C) II and IV D) I and III

D Issuers of municipal or municipal fund securities are exempt issuers and are not regulated or under the guidance of the MSRB or any other self-regulatory organization.

There are certain securities offerings that are limited to those who meet the definition of accredited investor. The SEC requires that the issuer shall take reasonable steps to verify that purchasers of securities sold in those offerings are accredited investors. One way in which this may be accomplished for natural persons is obtaining a written confirmation from certain persons or entities that such person or entity has taken reasonable steps to verify that the purchaser is an accredited investor within the prior three months and has determined that such purchaser is an accredited investor. Confirmation from which of the following would not meet the SEC's requirements? A) A registered broker-dealer B) A licensed attorney who is in good standing under the laws of the jurisdictions in which he or she is admitted to practice law C) A certified public accountant who is duly registered and in good standing under the laws of the place of his or her residence or principal office D) An investment adviser registered and in good standing under the laws of the state of its principal office

D It is only investment advisers registered with the SEC, not the state(s), for whom the written confirmation of their accredited investor status is acceptable.

An investor would most likely purchase money market instruments for their A) yields. B) appreciation potential. C) inflation protection. D) liquidity.

D Money market instruments are frequently referred to as cash equivalents. That is largely due to their high liquidity. Yields on these instruments are very low, and as fixed-income instruments, they offer no appreciation potential or inflation protection.

Which of the following is least important to a municipal bond analyst? A) Debt service to annual revenues B) Revenue collection record C) Tax collection ratio D) Legality of the issue

D Municipal bond analysts are concerned with the financial aspects of municipal bonds to ensure that they do not default. Various financial ratios and collection records are critical to their analysis. The legality of the municipal issue, as determined by the legal opinion, is important to issuers.

All of the following securities would be suitable investments for a traditional IRA except A) AAA rated U.S. government agency bonds. B) A rated corporate bonds. C) blue-chip common stocks. D) AAA rated municipal bonds.

D Municipal bonds, which generate tax-free interest income, are unsuitable for retirement plans. One loses the federally tax-free income at distribution.

One of your customers would like to purchase a government agency security for the UTMA account of her daughter. The daughter worked in construction over the summer and would like to use $1,275 of her savings for the purchase. Securities issued by which of these agencies could be purchased for this account? A) Federal Home Loan Mortgage Corporation B) Federal Farm Credit System C) Student Loan Marketing Association D) Federal National Mortgage Association

D Of this group, the only agency that would be able to sell $1,275 of securities is Fannie Mae. Their securities are available with a minimum denomination of $1,000 and then increments of $1. FHLMC also has the $1,000 initial minimum, but with $1,000 increments. The same numbers apply to the FCS, and Sallie Mae's minimum is $10,000. Another agency that would have met the investor's need is GNMA.

All of the following are money market instruments except A) commercial paper. B) bankers' acceptances. C) reverse repurchase agreements. D) options.

D Options are not money market instruments.

For tax-reporting purposes, qualified dividends are considered to be what type of income? A) Earned B) Passive C) Phantom D) Portfolio

D Portfolio income includes dividends (qualified or not), interest, and net capital gains derived from the sale of securities.

A limited partnership brought to market through a Rule 506(b) private placement may be sold to any of the following except A) an unlimited number of accredited investors. B) an investor with over $1 million net worth. C) 35 unaccredited investors. D) an unlimited number of unaccredited investors.

D Rule 506(c) is for accredited investors only, but unlike Rule 506(b), advertising is permitted.

Which two of the following statements regarding a customer's account profile are true? New account information must be sent to the customer for verification within 15 days of opening. New account information must be sent to the customer for verification within 30 days of opening. The customer's profile must be sent for updating and verification no less frequently than every 24 months. The customer's profile must be sent for updating and verification no less frequently than every 36 months. A) I and III B) I and IV C) II and III D) II and IV

D SEC Rule 17a-3 requires delivery of a copy of the account information within 30 days of opening (and every 36 months thereafter). Customers are to verify the information and note any relevant changes to the information.

An important feature of scheduled premium variable life insurance policies is that A) the cash value can never fall below the guaranteed minimum amount. B) better than expected performance of the separate account can lead to reduced premiums. C) purchasers must understand that there are no guarantees with these policies. D) the death benefit can never fall below the guaranteed minimum amount.

D Scheduled (fixed) premium variable life always has a guaranteed death benefit. Cash values cannot be guaranteed, only the death benefit. Better than expected performance of the separate account will lead to increased cash values, but it will not affect the premiums.

Which of the following is not considered a debt security? A) Promissory note B) Equipment trust certificate C) Debenture D) Prior lien preferred stock

D Stock, whether preferred or common, represents equity (ownership) and is never considered a debt security.

A customer purchases 2 QRS Jul 30 calls at 2 and 2 QRS Jul 30 puts at 2.50. She will break even when the price of the underlying stock is $25.50. $27.50. $32.00. $34.50. A) II and III B) III and IV C) I and II D) I and IV

D Straddles have two breakeven points: the strike price plus the sum of the two premiums, and the strike price minus the sum of the two premiums

corporation has an outstanding issue of 8% convertible debentures with a conversion price of $25. The bond indenture contains an antidilutive clause guaranteeing the debt holders the right to maintain proportionate equity conversion in the corporation. If the company pays a 10% stock dividend to its common shareholders, how will that affect the debenture holders? A) The interest rate on the debentures will increase to 8.8%. B) They will receive four shares of the common stock. C) Each debenture holder will receive a check for $100. D) The bonds will now be convertible at approximately 22.73.

D The antidilutive provision means the debenture holders will be able to convert into an equivalent share value as before. With a conversion price of $25, the bond is convertible into 40 shares ($1,000 ÷ $25). After the 10% stock dividend, they should be able to have 10% more shares, or 44 shares. That means the conversion price must be reduced. Divide $1,000 by 44 shares and the result is $22.73. Remember, anytime there is a stock dividend, prices go down.

A corporate bond is quoted in the Wall Street Journal as follows: Bid: 100½ Asked: 100¾ Bid Chg.: -⅛ YTM: 5.75% From this information, you know the nominal yield is A) 5.625%. B) less than 5.75%. C) 5.75%. D) greater than 5.75%.

D The bid and asked prices show that the bond is being quoted at a premium (above par), with a yield to maturity of 5.75%. When bonds are trading at a premium, the nominal yield (coupon rate) is greater than the yield to maturity.

KLP Corporation has extensive investments in the stocks and bonds of other corporations. Its portfolio income this year amounts to $700,000 in interest income from bonds and $400,000 in dividend income from common and preferred stock. On how much of its portfolio income must it pay taxes this year? A) $300,000. B) $1.1 million. C) $120,000. D) $820,000.

D The corporate exclusion is 70% of dividend income; therefore, KLP must pay taxes on all $700,000 of its interest income, but only 30% (or $120,000) of its dividend income, for a total of $820,000.

The coupon on a bond can be described as its A) yield to call. B) current yield. C) basis. D) nominal yield.

D The coupon on a bond is also known as the nominal yield, and it indicates the annual interest paid. For example, a 4% bond pays $40 of interest per year.

What is the amount of interest payable semiannually on a $1,000 par value, 5% corporate bond currently selling at 80 and redeemable at par in 20 years? A) $20 B) $50 C) $40 D) $25

D The interest is based on the par value of $1,000. The current market price is irrelevant. With a coupon rate of 5%, this bond pays $50 per year. That would be $25 semiannually

If interest rates increase, the interest payable on outstanding corporate bonds will A) increase. B) change according to the inverse payout theory. C) decrease. D) remain unchanged.

D The interest payable is the nominal yield, which is stated on the face of the bond. It is the percentage of face value the bond will pay each year regardless of the prevailing interest rates in the market. It is the market price of bonds, not the interest payable, that responds inversely to changes in interest rates.

Which of the following securities carries the highest degree of purchasing power risk? A) Blue-chip stock B) Convertible cumulative preferred stock C) Short-term note D) Long-term, high-grade bond

D The longer a fixed-income investment is held, the more vulnerable the investor is to purchasing power risk from inflation. Although preferred stock is also a fixed-income investment, convertible preferred will increase in value with the underlying common stock.

Transactions in all of the following are effected in the money market, as opposed to the capital market, except A) commercial paper. B) U.S. Treasury bills. C) jumbo CDs. D) municipal revenue bonds.

D The money market is the marketplace for short-term (less than one year) debt obligations. The capital market is where long-term capital is raised. Municipal bonds, being long term, are a part of the capital market.

FINRA Rule 2330 deals with a member's responsibility in the sale of certain insurance company-based products. Specifically the concern is with A) variable life insurance. B) immediate variable annuities. C) fixed-index annuities. D) deferred variable annuities.

D The subject of the FINRA rule is deferred variable annuities. It applies to the sale or exchange of this specific product.

Three months after that, the investor sells the shares received from the stock dividend at $112.57 per share. The tax consequence to the investor is A) $122.55 short-term capital gain. B) $245.10 short-term capital gain. C) $1,879.10 long-term capital gain. D) $245.10 long-term capital gain.

D The total value of the initial position is unchanged, remaining at $24,012 (200 times $120.06). After the stock dividend the investor owns 230 shares (200 times 15% = 30 + 200 = 230). Therefore, the adjusted cost basis is $104.40 per share ($24,012 divided by 230 = $104.40). The question tells us that the investor sells those 30 additional shares at $112.57 per share. That is a difference of $8.17 per share. Multiply that gain by 30 shares and the result is a profit of $245.10.

An investor purchased 200 shares of DCAST common stock at $200 per share. What is the adjusted cost basis per share of this position after the company pays a 100% stock dividend? A) $200 B) $400 C) $50 D) $100

D The total value of the initial position is unchanged, remaining at $40,000 (200 times $200). After the stock dividend, the investor owns 400 shares (200 times 100% = 200 + 200 = 400). Therefore, the adjusted cost basis is $100.00 per share ($40,000 divided by 400 = $100). Perhaps you recognized that a 100% stock dividend has the same effect as a 2:1 split. That is, the stock's cost basis is cut in half.

A bond with a 9% coupon, maturing in 18 years and 6 months, is selling at 120. The yield to maturity is closest to A) 7.50%. B) 11.66%. C) 9.00%. D) 7.05%.

D This bond is selling at a premium (120% of par). Therefore, all of the computed returns must be lower than the 9% nominal (coupon) yield. Only two of them are. The 7.50% represents the current yield ($90 ÷ $1,200) That means the nominal yield is the highest, followed by the current yield (CY), the yield to maturity (YTM), and finally the yield to call (YTC) as the lowest. Because only one choice is lower than the CY, you get the correct answer with minimal effort.

Before effecting a penny stock transaction with a customer, the member firm must A) provide the customer with the price of the most recent trade in the stock. B) receive the signed copy of the risk disclosure document. C) verify that the customer has sufficient funds in the account. D) provide the customer with a current bid and asked quote on the stock.

D To avoid price gouging, SEC Rule 15g-3 requires that no penny stock transaction may take place without the member firm providing the customer with the current inside market quotes

Debt instruments put up for auction by the U.S. Treasury that offer intermediate maturities best describes A) Treasury bonds. B) anticipation notes. C) Treasury bills. D) Treasury notes.

D Treasury notes are the intermediate maturity (2 to 10 years). Treasury bills are short term (less than one year), anticipation notes are short-term revenue notes, and Treasury bonds are long term (10 years or more).

Complying with the suitability rules involves evaluating all of the following except A) reasonable-basis suitability. B) quantitative suitability. C) customer-specific suitability. D) qualitative suitability.

D Under FINRA Rule 2111, there are three main obligations: reasonable-basis suitability, customer-specific suitability, and quantitative suitability. There is no such thing as qualitative suitability.

Three brothers open a joint account instructing you that if one of them dies, they want the cash and securities in the account to go to the remaining parties to the account. The account should be opened A) as community property. B) as tenants in common. C) as tenants by the entireties. D) with right of survivorship.

D Using tenants with right of survivorship, each brother's interest in the account would go to the surviving brothers.

A corporation has $25 million of 5% bonds outstanding. The bonds are callable at 102. Current market interest rates are 6%. If the company would like to retire $10 million of the debt, it might be smart to A) exercise the call provision for $10 million face amount of the bonds. B) issue $10 million of treasury stock and use the proceeds to retire the bonds. C) issue $10 million of new bonds at current rates and use the proceeds to call in outstanding ones. D) make a tender offer to purchase $10 million face amount of the bonds.

D When current market interest rates are 6%, bonds with a 5% coupon are selling at a discount. That means the company could make a public offer to buy the bonds back at a price somewhat below par value. In simple terms, they could retire $10 million of debt for less than $10 million. It would make no sense to call the bonds at 102 ($1,020) when they can be purchased for less than $1,000 each in the open market.

DMF Company has $50 million of convertible bonds (convertible at $50) outstanding. The current market value of DMF's stock is $42. The bond indenture contains a nondilution feature. If DMF declares a 10% stock dividend, the new conversion price will be A) higher than $50. B) the stock's current market price. C) $50. D) lower than $50.

D With an antidilution feature, the issuer will increase the number of shares available upon conversion if the company declares a stock split or stock dividend. This means the bondholder must be able to convert it to more shares, which requires a lower conversion price.

An investor purchased an XYZ Oct 50 call for a premium of 4. On the expiration date, XYZ is selling for 62, and the investor closes the position at the option's intrinsic value. For tax purposes, the investor has realized A) $1,200 of ordinary income. B) $800 of ordinary income. C) a $1,200 short-term capital gain. D) an $800 short-term capital gain.

D With the stock selling at 62, a 50 call has intrinsic value of 12 points (call-up rule). That would represent proceeds of $1,200 to the owner of the call. Subtracting the $400 cost results in a short-term capital gain of $800.

All of the following will affect the working capital of a corporation except A) declaration of a cash dividend. B) an increase in current assets. C) a decrease in current liabilities. D) payment of a cash dividend.

D Working capital is defined as current assets minus current liabilities. On the declaration date, the future dividend payment is "booked" as a current liability (dividend payable). When the payment date comes, disbursement of the cash dividend will reduce current assets (cash) and current liabilities (dividend payable) by the same amount, leaving working capital unchanged.

One of your customers purchased a fixed premium variable life insurance policy five years ago. The face value of the policy is $2 million and the current cash value is $107,237. The customer calls you and asks about taking a policy loan. Although the exact details are in the prospectus, you know that the minimum amount that could be borrowed is A) $107,237.00. B) $96,513.30. C) $1,500,000.00. D) $80,427.75.

D the policy must allow for a policy loan equal to a minimum of 75% of the current cash value.

The basis of a bond with a 5% nominal yield maturing in twenty years and selling at 115 is approximately A) 4.65%. B) 5.75%. C) 4.35%. D) 3.95%.

D Annual interest - (premium ÷ number of years to maturity) (Current market price + par) ÷ 2) Plugging in the numbers, we have: ($50 - $7.50) divided by $1,075. That is 3.95%

On Wednesday, April 22, 2020, your customer purchased a block of City Y 4% Recreation Authority term revenue bonds quoted at 22. The bond's stated interest payment dates are J/J 1. After receiving the confirmation, the customer called you and asked why there was no additional cost for accrued interest. The most likely reason for that is A) the trade settled on an interest payment date. B) the accrued interest is paid by the seller. C) there was a mistake on the confirmation and it will be rectified shortly. D) the bonds are trading flat.

D Bonds trading without accrued interest are trading flat. Every bond trades flat twice a year: when the bond settles on an interest payment date. However, that is not the case here because this trade would settle on April 24 and interest payment dates are January and July 1. The price is a hint. A 4% bond selling at 22% of par indicates that this bond is likely in default of interest and that is why it is trading flat.

A customer purchased a full faith and credit bond. This bond would be known as A) a sinking or surplus fund bond. B) a revenue bond. C) a moral obligation bond. D) a general obligation bond.

D General obligation bonds are also known as full faith and credit bonds.

On September 1, an investor sold 100 shares of KLP Corporation common stock for a loss of $1 per share. On September 15, he purchased a KLP convertible bond with a conversion price of $40. How much of the original loss may he now declare for tax purposes? A) $40 B) $100 C) None D) $75

D- Because he purchased the convertible bond less than 30 days after realizing the loss, the sale of the stock falls under the wash sale rule. Investors who sell a security at a loss, and repurchase it, including its equivalent (e.g., convertible bond, warrant, or call option), 30 days before or after the sale will have the loss disallowed by the IRS. With a conversion price of $40, the bond could be converted into 25 shares (1,000 / 40) of KLP common stock. Hence, the investor has "bought back" the equivalent of 25 shares and may only declare a $75 loss, as the remaining $25 loss will be disallowed. Look at this question as if it said, "On September 15, he purchased 25 shares of KLP stock." That washes out $25 of the loss, but the rest is okay.

Several investors open an account as tenants in common (TIC). For suitability purposes, financial information is required on which of the following investors? A. Most of the investors B. The largest investor only C. Only the one authorized to trade teh account D. All of the investors

D. When a join account is opened, to be able to make suitable recommendations, financial information should be obtained on all ofthe account owners.

Which of the following individuals may not open a joint account? A. Two spouses B. Three sisters C. Two friends D. Parent and a minor child

D. A joint account may be opened by two or more individuals who have legal standing. A minor may not be a party in a joint account because minors are not legally considered a person

An investor has a margin account with the following positions: Long 100 shares ABC, LMV is $40 per share and DR is $1,800 Long 200 shares DEF, LMV is $30 per share and DR is $3,100 Short 100 shares GHI, SMV is $50 per share and CR is $9,000 The combined equity in the account is A) $14,100. B) $1,100. C) $6,100. D) $9,100.

LMV + CR - DR - SMV = EQ. D we have long market value (LMV) of $10,000 ($4,000 + $6,000). To that we add the credit balance (CR) of $9,000, giving us a total of $19,000. From that, we subtract the debit balance of $4,900 ($1,800 + $3,100) plus the short market value (SMV) of $5,000, or a total of $9,900. $19,000 ‒ $9,900 = equity of $9,100.

Appeals

May be made to the NAC, within 25 day

In determining a violation of position limits, short calls are aggregated with A) long calls. B) long puts. C) short puts. D) all of these.

Position limits are measured by the number of contracts on the same side of the market. Long calls and short puts are on the bull side; short calls and long puts are on the bear side.

Joint Tenants with Rights of Survivorship (JTWROS) Account

Stipulates that a deceased tenants interest in the account passes to the surviving tenants. Ownership is equal. Undivided interest

C Corporation

The most common type of corporation, which is a legal business entity that offers limited liability to all of its owners, who are called stockholders. If a business expects to need significat capital, this form is almost always the preferred choice. C corp earnings are subject to double taxation.

Taxable distributions from a retirement account are taxed as

Ordinary Income

Limited Partnership

Typical case with DPPs.

General Partnership

Unincorporated business owned and operated by two or more persons The profits and losses of the business flow directly through to the investors as passive income (or loss) for tax purposes. Thus avoiding double taxation of profits at the business and individual levels.

Complaints

a complaint is only a complaint if it is in writing, and complaint records are retained for a minimum of four years.

Which of the following disclosures regarding a collateralized mortgage obligation (CMO) is accurate? A) All tranches have the same degree of risk and the same risk characteristics. B) CMOs are unique in that they are suitable for small or unsophisticated investors to invest in mortgage-backed securities. C) The rate of return may vary as a result of early prepayment. D) Repayment of principal is guaranteed.

c


Ensembles d'études connexes

Ch2: Life Insurance: Life Insurance - Life Basics

View Set

Flash Cards Ch 4 retirement and other insurance concepts

View Set

Chapter 15: Obsessive-Compulsive and Related Disorders

View Set

Biology Mid-Term: Cell Function and Structure

View Set